Está en la página 1de 55

Calculo de Probabilidades.

Enunciados.
25 de septiembre de 2006
2

Indice general
1. Espacio de probabilidad 5
1.1. Probabilidad . . . . . . . . . . . . . . . . . . . . . . . . . . . . . . . . . . . . . . 5
1.2. Probabilidad condicionada, teorema de Bayes e independencia . . . . . . . . . . . 8
2. Variables y vectores aleatorios 17
2.1. Variable aleatoria . . . . . . . . . . . . . . . . . . . . . . . . . . . . . . . . . . . . 17
2.2. Vector aleatorio . . . . . . . . . . . . . . . . . . . . . . . . . . . . . . . . . . . . . 21
2.3. Independencia de variables aleatorias . . . . . . . . . . . . . . . . . . . . . . . . . 21
2.4. Distribuciones condicionadas . . . . . . . . . . . . . . . . . . . . . . . . . . . . . 23
2.5. Funci on de una o varias variables aleatorias . . . . . . . . . . . . . . . . . . . . . 24
3. Esperanza 27
3.1. Esperanza de una variable aleatoria . . . . . . . . . . . . . . . . . . . . . . . . . . 27
3.2. Esperanza de un vector aleatorio . . . . . . . . . . . . . . . . . . . . . . . . . . . 32
3.3. Esperanza condicionada . . . . . . . . . . . . . . . . . . . . . . . . . . . . . . . . 35
4. Convergencia de sucesiones de variables aleatorias 39
4.1. Tipos de convergencia . . . . . . . . . . . . . . . . . . . . . . . . . . . . . . . . . 39
4.2. Leyes de los Grandes N umeros . . . . . . . . . . . . . . . . . . . . . . . . . . . . 40
4.3. Funci on caracterstica . . . . . . . . . . . . . . . . . . . . . . . . . . . . . . . . . 41
4.4. Teorema Central de Lmite . . . . . . . . . . . . . . . . . . . . . . . . . . . . . . 41
4.5. Funci on generatriz de momentos . . . . . . . . . . . . . . . . . . . . . . . . . . . 42
5. Examenes previos 43
5.1. 1 de septiembre de 2004 . . . . . . . . . . . . . . . . . . . . . . . . . . . . . . . . 43
5.1.1. Castellano . . . . . . . . . . . . . . . . . . . . . . . . . . . . . . . . . . . . 43
5.1.2. Valenciano . . . . . . . . . . . . . . . . . . . . . . . . . . . . . . . . . . . 44
5.2. 3 de febrero de 2004 . . . . . . . . . . . . . . . . . . . . . . . . . . . . . . . . . . 45
5.2.1. Castellano . . . . . . . . . . . . . . . . . . . . . . . . . . . . . . . . . . . . 45
5.2.2. Valenciano . . . . . . . . . . . . . . . . . . . . . . . . . . . . . . . . . . . 46
5.3. 3 de septiembre de 2005 . . . . . . . . . . . . . . . . . . . . . . . . . . . . . . . . 47
5.3.1. Castellano . . . . . . . . . . . . . . . . . . . . . . . . . . . . . . . . . . . . 47
5.3.2. Valenciano . . . . . . . . . . . . . . . . . . . . . . . . . . . . . . . . . . . 47
5.4. 8 de junio de 2004 . . . . . . . . . . . . . . . . . . . . . . . . . . . . . . . . . . . 48
5.4.1. Castellano . . . . . . . . . . . . . . . . . . . . . . . . . . . . . . . . . . . . 48
5.4.2. Valenciano . . . . . . . . . . . . . . . . . . . . . . . . . . . . . . . . . . . 49
5.5. 9 de febrero de 2005 . . . . . . . . . . . . . . . . . . . . . . . . . . . . . . . . . . 50
5.5.1. Castellano . . . . . . . . . . . . . . . . . . . . . . . . . . . . . . . . . . . . 50
3
4

INDICE GENERAL
5.5.2. Valenciano . . . . . . . . . . . . . . . . . . . . . . . . . . . . . . . . . . . 51
5.6. 21 de junio de 2005 . . . . . . . . . . . . . . . . . . . . . . . . . . . . . . . . . . . 51
5.6.1. Castellano . . . . . . . . . . . . . . . . . . . . . . . . . . . . . . . . . . . . 51
5.6.2. Valenciano . . . . . . . . . . . . . . . . . . . . . . . . . . . . . . . . . . . 52
5.7. 6 de junio de 2006 . . . . . . . . . . . . . . . . . . . . . . . . . . . . . . . . . . . 54
5.7.1. Castellano . . . . . . . . . . . . . . . . . . . . . . . . . . . . . . . . . . . . 54
5.7.2. Valenciano . . . . . . . . . . . . . . . . . . . . . . . . . . . . . . . . . . . 55
Captulo 1
Espacio de probabilidad
1.1. Probabilidad
Problema 1 Juego de dados tradicional chino que se juega durante la celebraci on del a no nue-
vo. En este juego se lanzan 6 dados. Seg un parece un lanzamiento con dos pares gana a un
lanzamiento con un par. Cu al es la probabilidad de cada uno de estos sucesos? En otras pala-
bras, encuentra la probabilidad de obtener un par en un lanzamiento de 6 dados y la probabilidad
de obtener dos pares en un lanzamiento de 6 dados.
Problema 2 (Problema de los cumplea nos) En una reuni on hay n personas. Cu al es la
probabilidad de que dos de ellas tengan el mismo cumplea nos?
Problema 3 (Pitman, pagina 9) Elegimos una palabra al azar de esta frase. Se pide:
1. Que probabilidad tenemos de que la palabra tenga al menos cuatro letras?
2. Y de que la palabra tenga al menos dos vocales?
3. Y de que tenga al menos dos letras y al menos dos vocales?
Problema 4 (Muestreo con y sin reemplazamiento) Veamos un experimento que corres-
ponde a lo que se conoce como muestreo con reemplazamiento Una caja contiene una serie de
papeletas marcadas con los n umeros 1, . . . , n. Elegimos al azar una papeleta de la caja. Vemos
su n umero y la devolvemos a la caja. Determinar las probabilidades de los siguientes sucesos.
1. La primera papeleta tiene el n umero 1 y la segunda el n umero 2.
2. Los n umeros de las dos papeletas son n umeros enteros consecutivos, esto es, la primera
papeleta tiene un n umero una unidad inferior a la segunda.
3. El segundo n umero extraido es mayor que el primero.
Supongamos ahora que no reemplazamos la primera papeleta en la caja. En consecuencia la
segunda papeleta ha de ser distinta a la primera. Se pide responder a las tres preguntas anteriores
en esta nueva situaci on.
Problema 5 Supongamos que barajamos una baraja de 52 cartas y tomamos las dos cartas que
han quedado en la parte superior del mazo.
5
6 CAP

ITULO 1. ESPACIO DE PROBABILIDAD


1. Cu antos pares ordenados de cartas podemos obtener como resultado? En lo que sigue
vamos a asumir que cada uno de estos pares tiene la misma probabilidad de producirse.
2. Que probabilidad tenemos de que la primera carta sea un as?
3. Que probabilidad tenemos de que la segunda carta sea un as?
4. Y de que ambas cartas sean ases?
5. Y de que al menos tengamos un as entre las dos cartas?
Problema 6 Tenemos diez puntos colocados de forma equidistante en la circunferencia de un
crculo y elegimos aleatoriamente tres de entre esos diez puntos. Se pide:
1. Si A y B son dos puntos particulares adyancentes, que probabilidad tenemos de que A y
B esten entre los puntos seleccionados?
2. Que probabilidad tenemos de que entre los tres puntos seleccionados aleatoriamente ten-
gamos como mnimo un par de puntos adyacentes?
Problema 7 Se dispone de n
1
cubos blancos y n
2
cubos rojos. Se los ordena al azar en com-
partimentos numerados de 1 a n
1
+n
2
.
1. Cu al es el n umero N de disposiciones distintas posibles?
2. Calcular la probabilidad de que K cubos blancos determinados se encuentren en lugares
jados.
Problema 8 (Examen 9-2-2005) Cu al es la probabilidad de que una mano de p oquer con-
tenga s olo una pareja?
Nota: una baraja de p oquer tiene cuatro palos y de cada palo hay 13 cartas. En una mano se
sirven cinco cartas.
Problema 9 (Poker) La baraja francesa consta de 52 cartas distribuidas en cuatro palos o
colores: treboles, diamantes, corazones y picas. Cada uno de estos palos est a compuesto por 13
cartas: uno o as, dos, tres, cuatro, cinco, seis, siete, ocho, nueve, diez y las tres guras, que
se llaman valet (V, equivalente al Bube alem an, al Jack ingles, e incluso puede asimilarse a
la Sota espa nola), Dame (D, equivalente a la Dame alemana y a la Queen inglesa) y Roi (R,
equivalente al K onig alem an, al King ingles, y tambien al Rey de la baraja espa nola). En una
mano de poker se reparten 5 cartas a cada jugador. Se pide hallar la probabilidad de cada uno
de estos sucesos:
1. Tener escalera de color (5 cartas consecutivas del mismo palo).
2. Tener poker (4 cartas iguales x x x x y).
3. Tener un full (un tro y una pareja x x x y y).
4. Tener 5 cartas del mismo palo.
5. Tener una escalera (5 cartas consecutivas).
6. Tener un tro (x x x y z)
7. Tener dobles parejas (x x y y z).
1.1. PROBABILIDAD 7
8. Tener una pareja (x x y z w).
Problema 10 Lanzamos dos dados. Determinar la probabilidad de los siguientes sucesos:
1. El m aximo de los dos valores que obtenemos es menor o igual a 2.
2. El m aximo de los dos valores es menor o igual a 3.
3. El m aximo de los dos n umeros es igual a 3.
4. Repite los dos apartados anteriores sustituyendo 3 por x donde x vara entre 1 y 6.
5. Si denotamos por p(x) con x = 1, . . . , 6 las probabilidades calculadas en el apartado ante-
rior comprueba que

6
i=1
p(x) = 1.
Problema 11 (Una carrera de tortugas) En la carrera de las grandes tortugas compiten
cuatro animales. Para darle un poco de animaci on a la carrera, los cuatro propietarios deciden
introducir los nombres de las tortugas en un sombrero y cada uno de los propietarios elige
aleatoriamente un nombre sin reemplazamiento. Cada propietario est a obligado a apostar por
la tortuga que le ha correspondido en el sorteo. Se pide:
1. Determinar la probabilidad de que todos los propietarios apuesten por sus tortugas.
2. Ning un propietario apueste por su propia tortuga.
3. El desafortunado propietario A elija a la tortuga Berzine que siempre pierde.
4. A apueste por la tortuga de B y B por la tortuga de A.
Problema 12 (Examen 2-3-2004) El holandes Christian Huygens public o en 1657 uno de
primeros libros sobre Probabilidad que se conocen, De Ratiociniis in Ludo Aleae (Del Razona-
miento en los Juegos de Azar), en el que planteaba una serie de problemas. El que se conoce
como segundo problema de Huygens lo enunciamos a continuaci on
Tres jugadores A, B y C participan en el siguiente juego. Una urna contiene a bolas blancas
y b negras. Los jugadores, en el orden ABCABC . . ., extraen una bola con reemplazamiento
hasta que uno de ellos obtiene una bola blanca y gana. Encontrar la probabilidad de ganar para
cada jugador.
Problema 13 Hemos cuadriculado una cierta zona en seis columnas y cuatro las. Denotamos
por C(i, j) la celda en la la i y columna j. Considerad el siguiente juego. Tenemos una cha
colocada en el rect angulo marcado con C(0, 0). La cha la movemos a la derecha o hacia arriba
desde la celda inicial C(4, 1) a la celda nal C(1, 6). Se pregunta:
1. Cu antos posibles caminos hay moverse desde C(4, 1) hasta C(1, 6)?
2. Si el jugador en su camino pasa por la celda C(2, 5) recibe un premio. Supongamos que
cada camino tiene la misma probabilidad, cu al es la probabilidad de que el jugador pase
por C(2, 5) en su camino de C(4, 1) a C(1, 6)?
Problema 14 (Aditividad nita y numerable) Demostrar que una medida de probabili-
dad es nitamente aditiva.
Problema 15 Comprobar que la denici on de probabilidad de Laplace verica los axiomas de
Kolmogorov. En denitiva, que es una medida de probabilidad.
8 CAP

ITULO 1. ESPACIO DE PROBABILIDAD


Problema 16 (Krief y Levy, pagina 81) Tenemos el espacio de probabilidad (, A, P). Se
pide:
1. Probar que si A, B y C son tres sucesos en este espacio se tiene que:
P(ABC) = P(A) +P(B) +P(C) P(AB) P(BC) P(AC) +P(ABC).
2. Sean A
1
, . . . , A
n
sucesos en (, A, P). Demostrar la desigualdad siguiente:
P(A
1
. . . A
n
)
n

i=1
P(A
i
).
En que caso la desigualdad anterior es una igualdad?
Problema 17 (Krief y Levy pagina 81) Sea (, A, P) un espacio de probabilidad. Se de-
nomina diferencia simetrica de dos sucesos A y B al suceso AB = (A B
c
) (A
c
B).
1. Probar que si tenemos los tres sucesos A, B y C entonces
P(AC) P(AB) +P(BC).
2. Para sucesos A, B, C y D se verica
P
_
(A B)(C D)
_
P(AC) +P(BD).
Problema 18 (Un camino aleatorio cule) El da 27 de julio de 1997 se celebraron elec-
ciones a la presidencia del Barca. Haba s olo dos candidatos, el se nor Fern andez y el se nor
N u nez, siendo este ultimo el ganador. Un socio con veleidades probabilsticas se hizo la siguien-
te pregunta: habr a ido el se nor N u nez por delante del se nor Fern andez a lo largo de todo el
escrutinio? El se nor N u nez obtuvo 24025 votos y el se nor Fern andez 5209.
1.2. Probabilidad condicionada, teorema de Bayes e inde-
pendencia
Problema 19 Sean A y B dos sucesos. Obtener la probabilidad de A B si ha ocurrido A o
si ha ocurrido A B. Comentar el resultado.
Problema 20 (La paradoja del caballero De Mere) En un juego consistente en lanzar
repetidamente un par de dados, encontrar el menor n umero n de lanzamientos para que la
probabilidad de obtener al menos un doble seis sea mayor que 0,5.
Comentarios
El origen de la paradoja est a en la pregunta que Antoine Gombauld, caballero De Mere, planteo a
Pascal Observaba De Mere una discrepancia entre la realidad, deducida de su larga experiencia
como jugador, y una antigua regla muy extendida entre los jugadores que armaba que n = 24.
Esta err onea regla tena su origen en la creencia de un comportamiento lineal de las probabi-
lidades. Se saba que si los lanzamientos eran de un solo dado y se persegua la obtenci on de
un seis, n = 4, pues p
3,1
= 0,4213 y p
4,1
= 0,5177. Se razonaba a continuaci on mediante una
sencilla regla de tres: 4 es a 6 como 24 a 36.
1.2. PROBABILIDAD CONDICIONADA, TEOREMA DE BAYES E INDEPENDENCIA 9
Problema 21 A y B juegan a un juego en el que A tiene una probabilidad p de ganar una
partida. El vencedor es aquel que gana dos partidas consecutivas. Encontrar el valor de p si se
sabe que cuando A pierde la primera partida, las probabilidades de ganar el juego para A y para
B son iguales.
Problema 22 (Jugando con un tetraedro) Tenemos un tetraedro un poco extra no. Una
cara es de color rojo, la otra de color azul y una tercera de color verde. La cuarta y ultima
cara tiene tres partes coloreadas respectivamente de rojo, azul y verde. Lanzamos el tetraedro y
nos jamos en el color de la cara en que se apoya el tetraedro. Consideramos los sucesos A
r
consistente en que se apoya en una cara con el color rojo. Denimos A
a
y A
v
anal agomente
sustituyendo el color rojo por el azul y el verde. Son independientes dos a dos los sucesos
aleatorios A
r
, A
a
y A
v
? Son independientes los tres sucesos?
Problema 23 Con objeto de estudiar la efectividad de un nuevo remedio contra el dolor de
cabeza una gran muestra formada por n personas ha sido seleccionada. Puesto que se piensa
que la reacci on de una persona al medicamento puede estar relacionada con el sexo, los datos
que se tomaron fueron especcos al sexo. La siguiente tabla resume los resultados obtenidos:
E E
c
H a b
M c d
donde E = { el medicamento ha sido efectivo }, E
c
= { el medicamento no ha sido efectivo },
H = { hombre }, M = { mujer } y a +b +c +d = n. Es decir, el n umero de mujeres para las
cuales ha sido efectivo el medicamento es c y as con el resto de la tabla.
1. Bas andose en estos datos, determinar (i) P(H), (ii) P(E), (iii) P(H | E), (iv) P(E | H).
2. Cuando son independientes los sucesos E y H? Demostrar que si ad bc = 0, E y M
son sucesos independientes.
3. Si E y M son independientes, decimos que el efecto medicamento y el factor sexo son
independientes. Por que podemos hacer esta armaci on?
Problema 24 Demostrar que si B
1
, . . . , B
n
es una partici on de B (es decir, son disjuntos y

n
i=1
B
i
= B), entonces
P(A | B) = P(A | B
1
)P(B
1
| B) +. . . +P(A | B
n
)P(B
n
| B).
Problema 25 Consideremos un espacio de probabilidad (, A, P). Se pide:
1. Demostrar que si A y B son sucesos independientes tambien lo son A y B
c
; A
c
y B; A
c
y B
c
.
2. Tres sucesos A, B y C se dicen independientes si las tres parejas (A, B), (A, C) y (B, C)
est an constituidas por sucesos independientes, y si
P(A B C) = P(A) P(B) P(C).
Se pide comprobar que las ternas (A
c
, B, C), (A
c
, B
c
, C) y (A
c
, B
c
, C
c
) son sucesos mu-
tuamente independientes si A, B y C lo son.
3. Demostrar, con un ejemplo, que aunque las parejas (A, B), (A, C) y (B, C) esten constitui-
das por sucesos independientes, no tiene porque ocurrir lo mismo con la terna (A, B, C).
10 CAP

ITULO 1. ESPACIO DE PROBABILIDAD


Problema 26 (Krief y Levy, pagina 82) Se consideran cuatro n umeros reales a, b, c y d
comprendidos entre 0 y 1. Determinar las condiciones necesarias y sucientes que deben vericar
estos cuatro n umeros para que se pueda denir un espacio probabilstico (, A, P) y dos sucesos
A y B en este espacio vericando que: P(A|B) = a, P(A|B
c
) = b, P(B|A) = c y P(B|A
c
) = d.
Problema 27 Hay sucesos que son independientes de s mismo. Por ejemplo, el suceso vaco
verica que P( ) = P() = 0 y por lo tanto es independiente de s mismo. Que ha de
vericar un suceso para que sea independiente de s mismo?
Problema 28 Dos jugadores A y B juegan a un juego en el que cada uno de ellos puede
efectuar n lanzamientos de dos dados, siendo A quien comienza. Las reglas del juego son las
siguientes:
Si A obtiene una suma 6 con los dados antes de que B haya obtenido una suma 7, A gana
el juego.
Si es B quien obtiene el 7 antes de que A haya obtenido el 6, es B quien gana.
El juego termina en empate cuando ambos han agotado su n lanzamientos.
Encontrar las expresiones de p
A
(n), p
B
(n) y p
E
(n) que denotan, respectivamente, que el ganador
es A, el ganador es B o el juego termina en empate. Calcular sus lmites cuando n .
Problema 29 Sean A y B dos sucesos incompatibles con probabilidad distinta de cero. Cu al
es la probabilidad de que A ocurra antes que B si el experimento se repite indenidamente?
Problema 30 (El juego de craps) Un jugador lanza dos dados, si la suma del primer lan-
zamiento es 7 u 11 gana, si la suma es 2, 3 o 12 pierde y si la suma es cualquier otro n umero
continua lanzando hasta que aparezca una suma 7 o la suma que inicialmente obtuvo. Si aparece
la suma 7 antes que la suma inicial pierde, en caso contrario gana. Calcular la probabilidad de
que gane el juego.
Problema 31 (El segundo problema de Huygens) El holandes Christian Huygens public o en
1657 uno de primeros libros sobre Probabilidad que se conocen, De Ratiociniis in Ludo Aleae
(Del Razonamiento en los Juegos de Azar), en el que planteaba una serie de problemas. El que
se conoce como segundo problema de Huygens lo enunciamos a continuaci on
Tres jugadores A, B y C participan en el siguiente juego. Una urna contiene a bolas blancas
y b negras. Los jugadores, en el orden ABCABC . . ., extraen una bola con reemplazamiento
hasta que uno de ellos obtiene una bola blanca y gana. Encontrar la probabilidad de ganar para
cada jugador.
Problema 32 (El problema de los puntos o del reparto de la apuesta) Dos jugadores
A y B juegan a un juego consistente en un n umero indeterminado de partidas. La probabilidad
de ganar en cada partida es p para A y 1p para B. Aquel de los dos que consigue antes vencer
en r partidas gana el juego y la apuesta que ambos hicieron. Si el juego se interrumpe antes de
nalizar, c omo se debe repartir la apuesta?
Problema 33 Utilizando argumentos probabilsticos, probar la igualdad (A > a)
1 +
Aa
A 1
+
(A a)(A a 1)
(A 1)(A2)
+
(A a) . . . 2 1
(A 1) . . . (a + 1)a
=
A
a
Sugerencia.- Una urna con A bolas de las cuales a son blancas, extracciones sucesivas sin
reemplazamiento, primera bola blanca, etc.
1.2. PROBABILIDAD CONDICIONADA, TEOREMA DE BAYES E INDEPENDENCIA11
Problema 34 ( ) Se lanza un dado una vez, si sale 1 se saca una bola de la urna I, si sale 2
o 3 se saca de la urna II, y en otro caso se saca de la urna III. La urna I tiene 5 bolas blancas,
3 verdes y 2 rojas; la urna II tiene 1 blanca 6 verdes y 3 rojas; la III tiene 3 blancas, 1 verde y
6 rojas. Determina las probabilidades siguientes:
1. que se elija una bola roja
2. que se haya seleccionado la urna II si ha salido roja.
Problema 35 Proporcionamos a A un trozo de papel para que escriba un signo + o un signo
, sabiendo que escribe el primero con probabilidad 1/3. El papel pasa a B, quien lo deja
como est a o cambia el signo antes de pasarlo a C. A continuaci on C, que puede o no haber
cambiado el signo, lo pasa a D, quien nalmente nos lo devuelve tras haber introducido o no
alg un nuevo cambio. Si comprobamos que el papel tiene escrito un signo + y sabemos que la
probabilidad de que B, C y D cambiaran el signo es 2/3, obtener la probabilidad de que A
escribiera originalmente un signo +.
Problema 36 Un aparato de diagn ostico autom atico emite un diagn ostico basado en el resul-
tado de n an alisis de un mismo paciente. Cada an alisis, independientemente de los restantes,
puede dar un resultado err oneo con probabilidad p. La probabilidad de un buen diagn ostico,
condicionada al n umero de an alisis correctos, es una funci on creciente de dicho n umero, g(m).
Durante una ma nana la m aquina ha diagnosticado a k pacientes. Encontrar la probabilidad
del suceso A ={al menos un paciente est a mal diagnosticado}. Particularizar el resultado para
g(m) = m/n.
Problema 37 Un taxi se ve involucrado en un accidente nocturno. En la ciudad hay dos
compa nas de taxis, los taxis Negros y los taxis Blancos. Se sabe que el 85 % de los taxis de
la ciudad son Negros y el 15 % restante son Blancos. Un testigo del accidente arma que el
taxi involucrado era Blanco y la abilidad de su testimonio es del 80 %, es decir, es capaz de
identicar correctamente el color del taxi el 80 % de las veces.
1. Sin ning un c alculo previo, piensas que es m as probable que el taxi accidentado fuera el
Negro o el Blanco?
2. Calcula la probabilidad de que el taxi accidentado fuera el Blanco y compara ambas res-
puestas.
3. Supongamos que para 0 p 1 el 100p% de los taxis son Blancos y que la abilidad
del testigo contin ua siendo del 80 %. Estudia la sensibilidad a los datos de la respuesta
anterior viendo como vara esta en funci on de p. A partir de que valor de p la probabilidad
de que haya sido el taxi Blanco el accidentado supera 0.5?
4. El an alisis anterior puede completarse permitiendo que la abilidad del testigo sea variable,
100q %, con 0 q 1. Determina la regi on dentro del cuadrado
{(p, q) : 0 p 1, 0 q 1}
en la que la probabilidad de que haya sido el taxi Blanco el accidentado supera 0.5.
Cuando en todo cuanto precede nos referimos a la probabilidad de que haya sido el taxi Blanco
se sobrentiende que dado que el testigo arma que era Blanco.
12 CAP

ITULO 1. ESPACIO DE PROBABILIDAD


Problema 38 (El coche y las cabras) En un concurso de TV hay tres puertas, una de ellas
esconde un coche y las otras dos sendas cabras. El concursante elige una de las puertas y obtiene
como premio aquello que la puerta oculta, pero la puerta permanece cerrada y el presentador,
que conoce lo que hay detr as de cada puerta, abre una de las otras dos puertas y aparece una
cabra (l ogicamente el presentador nunca abre la puerta que oculta el coche). El presentador
se dirige entonces al concursante y le permite cambiar su elecci on, que le conviene hacer al
concursante?
Este concurso tuvo gran exito a principios de los 90 en los USA y una conocida columnista
de la revista Parade Magazine, Marylin vos Savant public o que cambiando su elecci on el con-
cursante doblaba su probabilidad de ganar el coche, pues esta pasaba del 1/3 inicial a 2/3. Su
armaci on era correcta. Compruebalo.
Problema 39 El color de las ores de una cierta planta depende de dos genes, uno que recibe
del padre y el otro de la madre. Si los dos genes son identicos entonces la or tiene ese color;
sin embargo, con genes diferentes la or tiene bandas con cada uno de los dos colores. Los genes
presentes en la poblaci on corresponden a los colores azul, amarillo y verde y su proporci on en
la poblaci on es p, q y r (de modo que p + q + r = 1). Seleccionamos los padres de una planta
aleatoriamente dentro de la poblaci on y consideramos: el suceso A consistente en que las ores
del hijo tienen el color azul; el suceso B consistente en que las ores tengan m as de un color.
Se pide:
1. Determinar la probabilidad de los dos sucesos.
2. Demostrar que los dos sucesos son independientes si p =
2
3
y q = r =
1
6
.
3. Son estos los unicos valores de p, q y r que hacen a los sucesos A y B independientes?
Problema 40 Un test para diagnosticar cierta enfermedad tiene una sensibilidad del 95 % y
una especicidad del 99 %. Si la enfermedad en cuesti on tiene una prevalencia del 0.5 %, cu al
es el valor predictivo del test?
Problema 41 Supongamos una clase con n estudiantes. Uno de ellos conoce una historia sobre
Jesuln. Se la cuenta a uno de sus compa neros elegido al azar. A su vez este segundo estudiante
vuelve a contarla a otro compa nero diferente del que se la ha contado elegido al azar. El rumor
sigue propag andose de este modo. En cada ocasi on la persona cuenta la historia a otra del grupo
elegida al azar excluyendo a la persona que le ha informado. Cu al es la probabilidad de que la
historia se cuente k veces sin que se la cuenten dos veces al mismo individuo?
Sugerencia:denid los sucesos A
i
con i = 1, . . . , k consistentes en que la historia no se
repite la i-esima vez que se cuenta. A partir de estos sucesos denir el suceso de interes.
Problema 42 Un sistema compuesto por n componentes trabaja en paralelo si funciona cuando
al menos una componente funciona. Si la componente i funciona independendientemente de las
dem as con probabilidad p
i
para i = 1, . . . n , cu al es la probabilidad de que el sistema funcione?
Problema 43 Ha desaparecido un avi on y se cree que es igualmente probable que se encuentre
en cualquiera de las regiones R
1
, R
2
o R
3
. Sea 1
i
la probabilidad de que se encuentre el avi on
mientras se busca en la regi on i (en la pr actica estas probabilidades dependen de las condiciones
geogr acas y del entorno de las regiones). Cu al es la probabilidad de que el avi on se encuentre
en la regi on i dado que la b usqueda en la regi on 1 ha sido infructuosa?
Problema 44 Si se lanzan dos dados equilibrados
1.2. PROBABILIDAD CONDICIONADA, TEOREMA DE BAYES E INDEPENDENCIA13
1. cu al es la probabilidad de que al menos uno de ellos sea un 6 dado que los dos n umeros
que han salido son diferentes?
2. cu al es la probabilidad de que el primero de ellos sea un 6 sabiendo que la suma de los
dos n umeros que han salido es i? H allala para todos los valores de i entre 2 y 12.
Problema 45 Es el doble de probable desarrollar un embarazo ect opico para una embarazada
fumadora que para una embarazada no fumadora. Si el 32 % de las mujeres en edad fertil son
fumadoras, que porcentaje de mujeres con embarazos ect opicos son fumadoras?
Problema 46 Supongamos que el tiempo (seco o lluvioso) ma nana ser a el mismo que el de hoy
con probabilidad p. Si el tiempo el 1 de enero es seco, demuestra que P
n
, que es la probabilidad
de que sea seco n das despues, satisface:
P
n
= (2p 1)P
n1
+ (1 p), n 1
P
0
= 1.
Demuestra que
P
n
=
1
2
+
1
2
.(2p 1)
n
Problema 47 Tres prisioneros A, B y C son informados por su carcelero de que se ha elegido
al azar a uno de ellos para ser ejecutado y que los otros dos van a ser liberados. El prisionero A
le pide al carcelero que le diga en privado cu al de sus compa neros va a ser liberado, asegur andole
que no pasa nada porque le de esa informaci on puesto que el sabe que al menos uno de los otros
dos quedar a libre. El carcelero no quiere contestar la pregunta porque dice que si A supiera cu al
de sus dos compa neros va a ser liberado entones su propia probabilidad de ser ejecutado subira
de
1
3
a
1
2
porque entonces el sera uno de los dos que podra ser ejecutado. Que piensas del
razonamiento del carcelero?
Problema 48 A y B se enfrentan en duelo. Las reglas del duelo son que ambos tienen que
recoger el arma y disparar al otro simult aneamente. Si uno o ambos resultan heridos, el duelo
se acaba. Si ambos fallan repiten el proceso. Supongamos que los resultados de los disparos son
independientes y que un disparo de A alcanza a B con probabilidad p
A
y que un disparo de B
alcanza a A con probabilidad p
B
. Cu al es
1. la probabilidad de que A no resulte herido;
2. la probabilidad de que ambos duelistas resulten heridos;
3. la probabilidad de que el duelo acabe despues de n rondas de disparos;
4. la probabilidad de que el duelo acabe despues de n rondas de disparos dado que A no ha
sido herido;
5. la probabilidad de que el duelo acabe despues de n rondas de disparos dado que ambos
duelistas han sido heridos?
Problema 49 Supongamos que tenemos 10 monedas de manera que si se lanza la i-esima mo-
neda sale care cara con probabilidad
i
10
para i = 1, . . . , 10. Cuando se selecciona aleatoriamente
al azar una moneda y se lanza sale cara, cu al es la probabilidad de que la moneda seleccionada
fuese la quinta?
14 CAP

ITULO 1. ESPACIO DE PROBABILIDAD


Problema 50 Dos armarios en apariencia identicos tienen dos cajones. El armario A contiene
una moneda de plata en cada caj on y el armario B tiene una moneda de plata en un caj on y
una moneda de oro en el otro caj on. Se elige al azar un armario, se abre uno de los cajones y
se encuentra una moneda de plata, cu al es la probabilidad de que haya una moneda de oro en
el otro caj on?
Problema 51 Un modelo simplicado para el cambio del precio de una acci on en bolsa supone
que cada da el precio de la acci on aumenta 1 unidad con probabilidad p o baja 1 unidad con
probabilidad 1 p. Los cambios en das diferentes se consideran independientes.
1. Cu al es la probabilidad de que despues de dos das el precio sea el mismo?
2. Cu al es la probabilidad de que despues de dos das el precio haya aumentado en 1 unidad?
3. Dado que al cabo de tres das el precio de la acci on ha aumentado en 1 unidad, cu al es
la probabilidad de que subiera el primer da?
Problema 52 Una baraja de poker (52 cartas) se divide al azar en 4 montones de 13 cartas
cada uno. Calcula la probabilidad de que cada mont on contenga exactamente un as.
Ayuda Dene los sucesos E
i
, para i = 1, 2, 3, 4 como sigue y usa la regla de la multiplicaci on:
E
1
= {el as de picas est a en cualquiera de los montones}
E
2
= {el as de picas y el as de corazones est an en montones diferentes}
E
3
= {los ases de picas, corazones y diamantes est an en montones diferentes}
E
4
= {los 4 ases est an en montones diferentes}
Problema 53 Hay 12 bolas en una urna. Tres jugadores A, B y C extraen sucesvamente una
bola de la urna (primero A, despues B y a continuaci on C). El ganador es el primero que extrae
una bola blanca. Halla las probabilidades de ganar para cada jugador si
1. Cada bola se reemplaza despues de su extracci on.
2. Las bolas extraidas no se reintroducen en la urna.
Problema 54 La probabilidad de ganar en un lanzamiento de dados es p. A empieza y si falla
le pasa los dados a B, que intenta ganar en su turno. Contin uan tirando los dados sucesvamente
hasta que uno de ellos gana. Cu ales son las probabilidades de ganar de cada uno de ellos ? Y
si fueran k jugadores?
Problema 55 Se busca un paraguas que, con probabilidad
p
7
, se encuentra en cualquiera de
los siete pisos de un inmueble. Se han explorado en vano los seis primeros pisos. Cu al es la
probabilidad de que el paraguas se encuentre en el septimo piso?
Problema 56 (Krief y Levy, pagina 87) Una urna contiene bolas blancas y bolas negras.
Se efect ua una sucesi on de n extracciones en la urna. Supongamos que la probabilidad de que
la k-esima bola sacada sea blanca, cuando las k 1 bolas precedentes lo fueron, es igual a
1
k+1
.
Calcular la probabilidad de que las n primeras bolas sacadas sean blancas.
Problema 57 Una bola marcada puede estar en una cualquiera de las dos urnas que tenemos
disponibles, con probabilidades p y 1 p, respectivamente. La probabilidad de extraer la bola de
la urna en la que est a alojada es r (r = 1). Cu al es la mejor forma de utilizar n extracciones
con reemplazamiento, de cualquiera de las dos urnas, para que la probabilidad de extraer la bola
sea m axima?
1.2. PROBABILIDAD CONDICIONADA, TEOREMA DE BAYES E INDEPENDENCIA15
Problema 58 Disponemos de 3 cajas de 20 piezas cada una. El n umero de piezas que re unen
las condiciones de calidad exigidas son, respectivamente, 20, 15 y 10. De una de las cajas elegida
al azar se extrae una pieza que resulta ser buena. Se devuelve a la caja y se extrae una segunda
pieza que tambien resulta buena. Cu al es la probabilidad de que la caja elegida haya sido la
tercera?
Problema 59 Dos especies muy parecidas de champi nones (especies I y II) son difciles de
distinguir sin la ayuda de un microscopio. Un metodo de campo habitualmente utilizado consiste
en observar la presencia o ausencia de un anillo en el champi n on. El 90 % de los individuos de
la especie I y el 20 % de los de la especie II tienen el anillo. Se sabe tambien que en la zona en
que se est a trabajando el 70 % de los champi nones son de la especie I. Se pide:
1. Supongamos que el recolector encuentra un champi n on con un anillo y decide que es de
la especie I. Con que probabilidad est a en lo cierto?
2. Si todos los champi nones con anillo son clasicados como de la especie I y los que no
lo tienen como de la especie II, que proporci on de champi nones estar a correctamente
clasicado?
Problema 60 (Krief y Levy, pagina 85) Dos personas escriben al azar un n umero entero
de dos cifras (comprendidas entre 10 y 99).
1. Se repite la experiencia n veces y se supone que los resultados son mutuamente indepen-
dientes. Que probabilidad tenemos de que las dos personas escriban, una vez al menos,
el mismo n umero? Denotemos esta probabilidad por p(n).
2. Calcular p(100).
3. Cu antas veces hara falta repetir la experiencia para que p(n) sea igual al 0,99?
Problema 61 Un jugador dispone de nueve dados: dos dados de tipo A, tres dados de tipo B
y cuatro dados de tipo C. La tabla siguiente indica, para cada tipo de dado, el n umero de caras
que llevan el n umero i (con i = 1, . . . , 6).
1 2 3 4 5 6
A 2 1 0 1 1 1
B 2 2 0 1 0 1
C 2 2 0 2 0 0
El jugador elige al azar uno solo de estos dados y hace 421 en tres tiradas. Cu ales son las
probabilidades respectivas de que haya jugado con un dado del tipo A, del tipo B o del tipo C?
Problema 62 Se dispone de dos dados A y B. El dado A tiene cuatro caras rojas y dos caras
blancas. El dado B tiene dos caras rojas y cuatro caras blancas. Se lanza una moneda: si se
obtiene cruz se decide jugar unicamente con el dado A; si se obtiene cara se decide jugar
unicamente con el dado B. Se pide calcular:
1. La probabilidad de obtener roja.
2. La probabilidad de obtener roja la tercera tirada sabiendo que ya se ha obtenido ese color
en las dos primeras tiradas.
3. La probabilidad p
n
de haber utilizado el dado A sabiendo que se ha obtenido roja en las n
primeras tiradas.
16 CAP

ITULO 1. ESPACIO DE PROBABILIDAD


Problema 63 (El problema de las coincidencias) Supongamos que 4 invitados llegan a
una casa y dejan el sombrero en el vestbulo. Si a la salida los recuperan de modo aleatorio,
calcular la probabilidad de que ninguno de ellos reciba su propio sombrero. Resolver el mismo
problema suponiendo que en lugar de cuatro invitados tenemos n invitados.
Problema 64 Repetimos indenidamente una prueba en la que la probabilidad de exito es
siempre la misma, p, siendo los resultados de las pruebas independientes unos de otros (se trata
de una sucesi on de pruebas de Bernoulli). Obtener la probabilidad de que a exitos ocurran antes
que b fracasos.
Problema 65 (La paradoja de la urna vaca) Disponemos de una urna innitamente gran-
de y de una colecci on innita de bolas numeradas. Procedemos a depositar las bolas en la urna
de tres formas distintas.
1. A las 5 de la tarde menos 1 minuto introducimos las 10 primeras extrayendo la que lleva el
n umero 10 (supongamos que la introducci on y la sucesiva extracci on consumen un tiempo
0). A las 5 menos
1
2
minuto depositamos las 10 bolas siguientes y extraemos la que lleva
el n umero 20. A las 5 menos
1
4
las 10 siguientes extrayendo a continuaci on la que lleva
el n umero 30. Y as sucesivamente.
2. El segundo procedimiento es an alogo al anterior, pero las bolas que se extraen en cada en
ocasi on son las numeradas 1, 2, 3, .....
3. En el tercer procedimiento las bolas se introducen como en los dos anteriores pero en cada
decena la extracci on se efectua al azar.
Cuantas bolas habr a en la urna a las 5 de la tarde seg un el procedimiento empleado?
Problema 66 (Krief y Levy, pagina 88) Se considera un conjunto de N + 1 urnas nume-
radas. Cada urna contiene N bolas rojas o blancas. En concreto la urna k contiene k 1 bolas
blancas y N k + 1 bolas rojas. Se escoge al azar una urna y se toman n bolas devolviendo a
la urna cada bola extrada antes de sacar la siguiente.
1. Determinar la probabilidad de que todas las bolas extradas sean blancas.
2. Siempre en la hip otesis de extracciones con reeemplazamiento, determinar la probabilidad
de que la n + 1-esima bola extrada sea blanca sabiendo que las n bolas extradas ante-
riormente han sido blancas. Dar valores aproximados de estas probabilidades en el caso
en que N es grande.
Captulo 2
Variables y vectores aleatorios
2.1. Variable aleatoria
Problema 67 Los autobuses llegan a la estaci on de Salat a intervalos de 10 minutos empezando
desde las 12 : 00. Un hombre llega a la parada un n umero aleatorio de minutos X despues de
las 12 : 00 si la funci on de distribuci on de X es:
F(x) =
_
_
_
0 si x < 0
x
60
si 0 x 60
1 si x > 60
Cu al es la probabilidad de que espere menos de 5 minutos?
Problema 68 Comprueba que la funci on f
Y
(y) =
1
2
si y (1, 1) (0 en otro caso) es una
densidad de probabilidad de una variable aleatoria Y . Halla la funci on de distribuci on de dicha
variable aleatoria.
Problema 69 La funci on de probabilidad de una variable aleatoria X viene dada por f
X
(i) =
c

i
i!
para i = 0, 1, . . ., donde es una constante positiva. Halla P(X = 0) y P(X > 2)
Problema 70 Se eligen dos bolas al azar sin reemplazamiento de una urna que contiene 8
bolas blancas, 4 bolas negras y 2 de color naranja. Supongamos que ganamos 2 euros por cada
bola blanca elegida y perdemos 1 euro por cada bola blanca elegida. Sea X la variable que denota
nuestras ganancias. Halla la funci on de cuanta (o probabilidad) de X.
Problema 71 La funci on de distribuci on de X viene dada por:
F(x) =
_

_
0 si x < 0
x
4
si 0 x < 1
1
2
+
x1
4
si 1 x < 2
11
12
si 2 x < 3
1 si x 3
1. Halla P(X = i) para i = 1, 2, 3
17
18 CAP

ITULO 2. VARIABLES Y VECTORES ALEATORIOS


2. Halla P(
1
2
< X <
3
2
)
Problema 72 La funci on de distribuci on de X viene dada por:
F(x) =
_

_
0 si x < 0
1
2
si 0 x < 1
3
5
si 1 x < 2
4
5
si 2 x < 3
9
10
si 3 x < 3,5
1 si x 3,5
Halla la funci on de cuanta de X.
Problema 73 Un vendedor de enciclopedias ha concertado dos citas con dos posibles compra-
dores. Conseguir a vender una enciclopedia en la cita con el primer cliente con una probabilidad
de 0,3 y lo lograr a en la cita con el segundo con una probabilidad de 0,6 (independientemente
de lo que haya sucedido con el primero). Si vende la enciclopedia de lujo ingresa 1000 euros
mientras que si vende la normal ingresa 500 euros. Supongamos que es igualmente probable
que venda cualquiera de las dos enciclopedias. Halla la funci on de cuanta de X que representa
el total de los ingresos del vendedor.
Problema 74 Cinco n umeros distintos se distribuyen al azar entre 5 jugadores numerados del
1 al 5. Cuando dos jugadores comparan sus n umeros el que lleva el n umero m as alto es el
ganador. Inicialmente los jugadores 1 y 2 comparan sus n umeros, el ganador se compara con
el jugador 3, y as sucesivamente. Denotamos por X el n umero de veces que gana el jugador 1.
Halla P(X = i) para i = 1, 2, 3, 4.
Problema 75 Una moneda no correcta con probabilidad de cara p y probabilidad de cruz 1 p
es lanzada hasta que aparece una cara o tres veces, lo que ocurra antes. Si X es el n umero de
lanzamientos que se realizan, se pide determinar la distribuci on de X.
Problema 76 (Examen 9-2-2005) Probar que para cualquier funci on de densidad de proba-
bilidad se verica
lm
x+
x
_
+
x
1
z
f(z)dz = 0.
Problema 77 Para que valores de la constante C las funciones siguientes son funciones de
cuanta sobre los enteros positivos?
1. Geometrica f(x) = C2
x
2. Logartmica f(x) = C
2
x
x
3. Inversa cuadrado f(x) =
C
x
2
4. Poisson modicada f(x) = C
2
x
x!
2.1. VARIABLE ALEATORIA 19
Problema 78 El responsable de una tienda de electr onica compra cierta clase de piezas en
lotes de tama no 10. Su poltica consiste en inspeccionar 3 al azar de cada lote y aceptarlo s olo
si las 3 funcionan correctamente. Si una quinta parte de los lotes contiene 4 piezas defectuosas
y los dem as s olo una pieza defectuosa, que proporci on de lotes rechazar a?
Problema 79 Para una distribuci on hipergeometrica halla
P(X=k+1)
P(X=k)
. Cu al es la moda de una
H(N, n, r)?
Problema 80 (Examen 9-2-2005) Una urna contiene n papeletas numeradas de 1 a n in-
clusive. Extraemos r al azar. Sea X el n umero mayor obtenido si las papeletas se reemplazan
despues de cada extracci on y sea Y el n umero mayor si las papeletas no se reemplazan en la
urna. Determinar las funciones de distribuci on, las funciones de cuanta (o probabilidad) y
demostrar que
F
Y
(k) < F
X
(k) para 0 < k < n. (2.1)
Problema 81 En un proceso de fabricaci on de hilados se producen roturas del hilo de manera
aleatoria a lo largo del tiempo. Es importante conocer cuando y c omo pueden producirse dichas
roturas. Supongamos que un trabajador controla 800 husos y que la probabilidad de rotura del
hilo en cada bobina, durante un cierto intervalo de tiempo , es p = 0,005. Encontrar el n umero
de roturas m as probable y la probabilidad de que se produzcan a lo sumo 10 roturas.
Problema 82 Samuel Pepy, contempor aneo de Isaac Newton, saba que al lanzar 6n dados el
n umero esperado de seises era n. A partir de este resultado deduca que los sucesos A
n
={al
menos n seises al lanzar 6n dados}, n = 1, 2, 3, tenan todos igual probabilidad. Isaac Newton
hubo de sacarlo de su error.
1
Problema 83 Sea X el n umero de pruebas de Bernoulli necesarias para obtener un exito y un
fracaso. Determinar la distribuci on de probabilidad de X.
Problema 84 Una moneda de 1 cm de di ametro se lanza y cae dentro de una lata cilndrica
cuyo fondo tiene 5 cm de di ametro (la moneda cae plana, no de canto).
1. Cu al es la probabilidad de que la moneda cubra el centro del fondo de la lata?
2. Supongamos que en lugar de usar una lata cilndrica se tira en una caja cuyo fondo es un
cuadrado cuyos lados miden 5 cm, cu al es ahora la probabilidad de que la moneda cubra
el centro del fondo de la lata?
Problema 85 Un testigo experto en un juicio sobre una supuesta paternidad testica que la
longitud en das de un embarazo (es decir desde el momento de la concepci on hasta el momento
del parto) se distribuye aproximadamente seg un una Normal con par ametros = 270 y = 10.
El presunto padre puede demostrar que estuvo fuera del pas durante un perodo de tiempo que
empezaba 290 das antes del nacimiento del ni no y que acababa 240 das antes del nacimiento.
Si el acusado fuese realmente el padre de la criatura, y suponiendo que es verdad lo que asegura
el experto cu al sera la probabilidad de que la madre tuviera un embarazo tan largo o tan corto
?
1
El problema, que es de f acil soluci on y puede incluso parecer ingenuo a alg un lector, se recoge aqu por su
interes hist orico y tambien porque el autor de esta colecci on ha tenido ocasi on de comprobar que los emulos
actuales de Samuel Pepy son todava numerosos
20 CAP

ITULO 2. VARIABLES Y VECTORES ALEATORIOS


Problema 86 La mediana de una variable aleatoria contnua con funci on de distribuci on F
es aquel valor m tal que F(m) =
1
2
. Es decir es igual de probable que una variable aleatoria sea
mayor que su mediana como que sea menor que ella.
La moda de una variable aleatoria contnua con funci on de densidad f es el valor de x para
el que f(x) alcanza su m aximo.
Halla la mediana y la moda de X si X se distribuye
1. U(a, b)
2. N(, )
3. Exp()
Problema 87 Un bit es trasmitido repitiendolo n veces. El mensaje es interpretado asignando
el valor que m as veces se recibe. Por ejemplo: si n = 5 y el mensaje recibido es 10010 entonces
concluimos que se envi o un 0 (se repite 3 veces frente a las dos veces que se repite el 1).
Suponiendo que n es un n umero impar y que cada bit del mensaje es transmitido correctamente
con probabilidad p, independientemente de los dem as bits, determina la probabilidad de que el
mensaje sea recibido correctamente (se reciba el bit que se trasmiti o).
Problema 88 Consideremos la distribuci on Beta con par ametros a y b. Demuestra que
1. cuando a > 1 y b > 1, la densidad es unimodal, es decir tiene una unica moda que es
m =
a1
a+b2
;
2. cuando a 1, b 1 y a +b < 2, la densidad es o bien unimodal con moda en 0 o en 1 o
bien tiene forma de U con modas tanto en 0 como en 1;
3. cuando a = 1 = b, todos los puntos de [0, 1] son modas.
Problema 89 Un fabricante de bolas para rodamientos somete su producto al siguiente proceso
de control de calidad. Las bolas son aceptadas si no pasan a traves de un agujero de di ametro
d
1
, pero s lo hacen a traves de otro de di ametro d
2
, d
2
> d
1
. Se sabe que el di ametro D de las
bolas es aleatorio con una distribuci on N(,
2
), = (d
1
+ d
2
)/2 y = (d
2
d
1
)/4. Cu al es
la probabilidad de rechazar una bola?
Problema 90 El tiempo que tardan en ser atendidos los clientes del servicio de caja de cierta
sucursal bancaria es una variable aleatoria T Exp(), con = 0,2. Durante una ma nana
han llegado 10 clientes, cu al es la probabilidad de que a lo sumo 3 de ellos hayan tardado m as
de 6 minutos en ser atendidos? (Suponemos que los clientes son atendidos independientemente
unos de otros).
Problema 91 (Los sorteos de La Primitiva) Un asiduo de La Primitiva anda un tanto
preocupado al comprobar que en los 18 ultimos sorteos hay algunos n umeros que no han sido
extrados. Piensa que las 108 extracciones (18 6) suponen un poco m as del doble de 49 y
que por tanto cada n umero debera haber aparecido, aproximadamente, unas dos veces. Y si el
sorteo no fuera correcto? Habr a n umeros m as probables que otros?
Problema 92 Un comerciante vende semillas en paquetes de 50. Supongamos que cada semilla
germina con una probabilidad de 0,99 independientemente de las dem as. El comerciante promete
cambiar al comprador cualquier paquete que contenga 3 o m as semillas que no germinen. Cu al
es la probabilidad de que el comerciante tenga que cambiar m as de 40 paquetes de los 4000 que
ha vendido?
2.2. VECTOR ALEATORIO 21
Problema 93 En una ciudad se va a someter a tratamiento a los ni nos de seis a nos m as
bajos que cierta talla. En esta ciudad hay 6580 ni nos de esta edad, y su estatura sigue una
distribuci on normal de 119 cm de media y 4 cm de desviaci on tpica. Se va a realizar una
campa na informativa indicando una talla por debajo de la cual se ha de tratar a los ni nos. Los
servicios de endocrinologa de la ciudad solo pueden atender a 750 ni nos. Se pregunta:
1. Que talla en cm deber a indicar la campa na?
2. En cuantos ni nos se ver a desbordado el servicio de endocrinologa si la campa na indica
por error un cm m as?
2.2. Vector aleatorio
Problema 94 Lanzamos tres veces consecutivas una moneda y denimos las variables alea-
torias X ={n umero de caras en los dos primeros lanzamientos} e Y ={n umero de caras en
los dos ultimos lanzamientos}. Obtener la distribuci on de probabilidad conjunta de X e Y , sus
marginales y el coeciente de correlaci on entre ambas.
Problema 95 Sean X
1
, X
2
, . . . , X
n
variables aleatorias con funci on de distribuci on conjunta
F y con funciones de distribuci on marginales F
1
, F
2
, . . . , F
n
. Demostrar que
1
n

i=1
[1 F
i
(x
i
)] F(x
1
, x
2
, . . . , x
n
) mn
1in
F
i
(x
i
).
Problema 96 Los 4 tipos de sangre principales se presentan en la poblaci on de los EEUU de
acuerdo con los siguientes porcentajes:
Tipo A B AB 0
Porcentaje 42 % 10 % 4 % 44
1. Si se eligen al azar a dos personas de esta poblaci on, cu al es la probabilidad de que su
sangre sea del mismo tipo?
2. Si se eligen cuatro personas al azar, sea P(k) la probabilidad de que haya exactamente k
tipos sanguneos diferentes. Halla P(k) para k = 1, 2, 3 y 4.
Problema 97 Escogemos aleatoriamente un punto del interior de un disco de radio R. Sea X
la distancia del punto elegido al centro del disco. Halla la funci on de distribuci on de X.
2.3. Independencia de variables aleatorias
Problema 98 Consideremos el espacio de probabilidad (, A, P) y sean A
1
y A
2
dos sucesos.
Denimos X
1
= 1
A1
y X
2
= 1
A2
, las funciones caractersticas asociadas. Demostrar que X
1
y
X
2
son independientes si y s olo si A
1
y A
2
lo son.
Problema 99 Dos personas lanzan, cada una de ellas, n veces una moneda. Obtener la pro-
babilidad de que ambas obtengan el mismo n umero de caras.
Problema 100 Tenemos dos lneas de comunicaci on telef onica paralelas de longitud l, sepa-
radas por una distancia d < l. Sabemos que, al azar y de manera independiente, se producen
sendas roturas a lo largo del recorrido de cada una de ellas. Encontrar la probabilidad de que la
distancia R entre ambas roturas no sea superior a c.
22 CAP

ITULO 2. VARIABLES Y VECTORES ALEATORIOS


Problema 101 Elegimos al azar dos puntos, X e Y , en el intervalo [0, a]. Calcular la funci on
de distribuci on de la distancia entre ellos.
Problema 102 Sea {X
k
}
k1
una colecci on de variables aleatorias independientes U(0,1). Sea
0 < x < 1, denimos N = mn{n 1 : X
1
+X
2
+ +X
n
> x}. Encontrar P(N > n).
Problema 103 Se escogen al azar dos n umeros a y b, a en el intervalo [1, 3] y b en el intervalo
[1, 1] . Halla la probabilidad de que la ecuaci on x
2
+ax +b = 0 tenga dos races reales.
Problema 104 Escogemos dos n umeros al azar entre 0 y 1.Cu al es la probabilidad de que el
primero sea mayor o igual que el cuadrado del segundo y al mismo tiempo que el segundo sea
mayor o igual que el cuadrado del primero?
Problema 105 Tenemos una urna con 12 bolas numeradas de 1 a 12. Extraemos dos bolas y
denotamos por X
1
y X
2
los valores que observamos en la primera y en la segunda extracci on.
Sea X la variable denida como el m aximo de las dos extracciones. Se pide:
1. Determinar la funci on de distribuci on de la variable X si suponemos que las dos extrac-
ciones se realizan con reemplazamiento.
2. Determinar la funci on de distribuci on de la variable X si suponemos que no hay reem-
plazamiento entre las dos extracciones.
Problema 106 (Examen 11-2-2000) Las puntuaciones obtenidas por los estudiantes del cen-
tro A en las pruebas de selectividad se distribuyen N( = 6,25,
2
= 1), mientras que las de los
estudiantes del centro se distribuyen N(6, 1,5). Elegimos al azar 2 estudiantes del centro A y 3
del centro B. Se pide:
1. la probabilidad de que la puntuaci on media de los 2 estudiantes de A sea superior a la
puntuaci on media de los 3 de B, y
2. la probabilidad de que al escoger al azar uno de los 5 estudiantes, su nota sea superior a
6,5.
Problema 107 Sean {X
k
, k = 1, . . . , n} variables aleatorias i. i. d. con distribuci on U(0, 1).
Tomemos 0 < x < 1 y denamos
N = mn{n 1; X
1
+X
2
+ +X
n
> x}.
Encontrar P(N > n).
Problema 108 Un grupo de 10 personas han quedado para comer entre las 12 : 00 y las 12 : 15.
Cada persona llega al restaurante independientemente de las dem as y seg un una distribuci on
uniforme en el intervalo de tiempo anterior.
1. Eva y Mara son dos miembros del grupo. Halla la probabilidad de que Eva llegue al menos
dos minutos antes que Mara.
2. Halla la probabilidad de que la primera persona en llegar aparezca antes de las 12 : 05 y
de que la ultima llegue despues de las 12 : 10.
Problema 109 Inyectamos a unos ratones microorganismos del tipo A y B en igual proporci on.
Se supone que los microorganismos efectivos de cada tipo se distribuyen independientemente con
arreglo a una misma distribuci on de Poisson de par ametro . Un rat on sobrevive si y s olo si
no hay microorganismos efectivos en la dosis inyectada. A los ratones muertos se les examina
para ver si contenan microorganismos de uno o de los dos tipos. Encontrar la probabilidad de
que un rat on muerto contenga microorganismos de un solo tipo.
2.4. DISTRIBUCIONES CONDICIONADAS 23
2.4. Distribuciones condicionadas
Problema 110 Elegimos un n umero aleatorio X de la siguiente forma: lanzamos repetidamen-
te una moneda hasta que nos muestre la primera cara, si el n umero lanzamientos que hemos
necesitado es N, elegimos aleatoriamente un entero k en 1, 2, . . . , 10
N
, valor que asignamos a
X. Encontrar la distribuci on de probabilidad de X.
Problema 111 Sobre un crculo cuyo radio R es aleatorio con funci on de densidad
f
R
(r) =
_

_
r
2
9
, r [0, 3];
0 en el resto,
elegimos un punto al azar. Si X designa la distancia del punto al origen, obtener la funci on de
distribuci on y la funci on de densidad de X|R = r.
Problema 112 Consideremos la siguiente funci on
f
X|Y
(x|y) =
_
y
x
e
y
x!
, si y 0, x = 0, 1, . . .
0, en el resto.
a) Demostrar que para cada y jo, f
X|Y
(x|y) es una funci on de probabilidad (la de X condi-
cionada a Y = y).
b) Si Y Exp(), con = 1, encontrar la densidad conjunta de X e Y .
c) Comprobar que la marginal de X viene dada por
f
X
(x) =
_
_
_
1
2
x+1
, x = 0, 1, . . .
0, en el resto.
Problema 113 Las variables aleatorias X e Y tienen una distribuci on conjunta uniforme
en el interior del tri angulo T de vertices (0,0), (2,0) y (1,2). Calcular P(X < 1, Y < 1) y
P(Y < 1|X < 1,5).
Problema 114 Calcular la probabilidad de poder formar un tri angulo con dos puntos elegidos
en el intervalo [0, 1] seg un los distintos metodos que se enumeran a continuaci on.
1. Los dos puntos se eligen al azar en el intervalo.
2. Elegimos primero un punto al azar y a continuaci on elegimos el segundo punto, tambien
al azar, pero sobre el trozo mayor.
3. Elegimos un punto al azar y a continuaci on uno de los trozos, elegido al azar, lo dividimos
en dos partes iguales. Calcular, para este metodo, la probabilidad de que el tri angulo sea
obtuso.
Problema 115 (Examen 21-6-2005) En una urna hay una bola roja. Extraemos tres cartas
de una baraja francesa (52 cartas repartidas en 4 palos) y a nadimos a la urna tantas bolas verdes
como ases hayamos extrado. A continuaci on lanzamos 2 veces una moneda cuya probabilidad
de cara es p = 1/5 y a nadimos tantas bolas rojas como cruces hayamos obtenido. Finalmente
llevamos a cabo 2 extracciones con reemplazamiento de la urna. Si X es el n umero de bolas
verdes a nadidas a la urna e Y el n umero de bolas rojas a nadidas a la urna,
24 CAP

ITULO 2. VARIABLES Y VECTORES ALEATORIOS


1. Obtener la funci on de probabilidad de X.
2. Obtener la funci on de probabilidad de Y .
3. Si las dos bolas extradas con reemplazamiento son rojas, cu al es la probabilidad de no
haber obtenido ning un as al extraer las 3 cartas de la baraja francesa?
2.5. Funcion de una o varias variables aleatorias
Problema 116 Sea X una variable aleatoria cuya densidad viene dada por
f
X
(x) =
_

_
0, si x < 0,
1
2
, si 0 x 1,
1
2x
2
, si x > 1.
Encontrar la densidad de Y = 1/X.
Problema 117 Se traza aleatoriamente una linea recta a traves del punto (0, l). Encontrar la
densidad de probabilidad de la abcisa en el origen, X, de dicha recta.
Problema 118 Sea el n umero de celulas por unidad de area en una preparaci on celular. Pue-
de demostrarse que la distancia, D, de una celula a su vecino m as pr oximo tiene por densidad
de probabilidad,
f
D
(d) =
_
2de
d
2
, d > 0
0, en el resto.
El area del mayor disco libre (sin contactos) centrado en una celula es A = D
2
. Encontrar la
densidad de probabilidad de A.
Problema 119 Las variables X e Y son independientes y continuas. Sea U una variable di-
cot omica, independiente de ambas con P(U = 1) = P(U = 1) = 1/2. Denimos S = UX y
T = UY . Comprobar que S y T son dependientes, pero S
2
y T
2
son independientes.
Problema 120 Sean X e Y variables aleatorias independientes tales que X Gamma(
1
, )
e Y Gamma(
2
, ). Demostrar que las variables X+Y y X/(X+Y ) son variables aleatorias
independientes.
Problema 121 Tenemos X
1
, . . . , X
n
variables aleatorias independientes tales que X
j
tiene
una distribuci on binomial negativa con par ametros r
j
y p con j = 1, . . . , n. Demostrar que
S
n
=

n
j=1
X
j
tiene una distribuci on binomial negativa con par ametros

n
j=1
r
j
y p.
Problema 122 Tenemos X
1
, . . . , X
n
variables aleatorias independientes tales que X
j
tiene
una distribuci on geometrica con par ametro p
j
. Sea N
n
= mn{X
1
, . . . , X
n
}. Demostrar que la
variable N
n
tiene una distribuci on geometrica con par ametro
p = 1
n

j=1
(1 p
j
). (2.2)
2.5. FUNCI

ON DE UNA O VARIAS VARIABLES ALEATORIAS 25


Problema 123 (Rohatgi, pagina 211) Sean X
1
, X
2
, . . . una sucesi on de variables aleatorias
independientes e identicamente distribuidas (i.i.d.) con distribuci on exponencial con par ametro
. Sea S
n
=

n
k=1
X
k
la n-esima suma parcial, con n = 1, 2, . . . y supongamos t > 0. Si Y es
el n umero de valores de S
n
en el intervalo [0, t] entonces Y sigue una distribuci on de Poisson
con par ametro t/.
Problema 124 (Examen 3-9-2005) Cuando una corriente de I amperios pasa a traves de
una resistencia de R ohmios, la potencia generada viene dada por W = I
2
R vatios. Supongamos
que I y R son variables aleatorias independientes con densidades
f
I
(x) =
_
_
_
6x(1 x), si 0 x 1;
0, fuera.
y
f
R
(y) =
_
_
_
2y, si 0 y 1;
0, fuera.
Hallar la densidad de W.
Problema 125 Demostrar que si tenemos variables aleatorias independientes X
1
, . . . , X
n
tales
que X
j
Gamma(
j
, ) con j = 1, . . . , n entonces S
n
=

n
j=1
X
j
Gamma(

n
j=1

j
, ).
Problema 126 (Rohatgi, pagina 213) Si X e Y son variables aleatorias exponenciales con
par ametro entonces Z = X/(X +Y ) sigue una distribuci on uniforme en el intervalo [0, 1].
Problema 127 (Rohatgi, pagina 215) Sean X e Y variables aleatorias independientes con
distribuci on Gamma(
1
, ) y Gamma(
2
, ) respectivamente. Demostrar que X/(X+Y ) sigue
una distribuci on beta con par ametros
1
y
2
.
Problema 128 (Examen 8-6-2004) Consideremos el siguiente procedimiento:
1. Generamos U con distribuci on uniforme en [0, 1].
2. Tomamos Y =
1

ln(1 U) siendo ln el logaritmo neperiano y una constante positiva.


3. Tomamos X = [Y ] donde [Y ] es la parte entera por exceso de Y .
Se pide:
1. La distribuci on de probabilidad de la variable aleatoria Y .
2. La funci on de probabilidad de la variable X. Comprobar que tiene una distribuci on geo-
metrica.
Problema 129 Sean X, Y, Z variables aleatorias i. i. d. con distribuci on U(0, 1). Encontrar la
densidad conjunta de XY y Z
2
y calcular P(XY Z
2
).
26 CAP

ITULO 2. VARIABLES Y VECTORES ALEATORIOS


Captulo 3
Esperanza
3.1. Esperanza de una variable aleatoria
Problema 130 Dos jugadores A y B juegan al siguiente juego: lanzan una moneda hasta que
aparece una cara. Si la cara aparece en el k-esimo lanzamiento el jugador B paga al jugador A
k monedas. Cuantas monedas deber a pagar el jugador A antes de iniciarse el juego para que
este sea equilibrado?
Problema 131 En dos lados distintos de un cuadrado unidad elegimos al azar e independien-
temente los puntos X e Y . Si por D designamos la distancia entre ellos, calcular E(D
2
) en las
dos situaciones posibles descritas en la gura.
Y
X
Y
X
D
D
Problema 132 Sean X e Y variables aleatorias independientes. Demostrar que si X y XY
son independientes, entonces X es una variable aleatoria degenerada.
Problema 133 Sea X una variable aleatoria discreta con soporte D
X
= {x
1
, x
2
, . . . , x
m
}.
Calcular
lm
n
E(X
n+1
)
E(X
n
)
.
Problema 134 Dos puntos se eligen al azar sobre un segmento de longitud a. Encontrar la
esperanza y la varianza de la distancia entre ellos.
Problema 135 (Si hay cumplea nos no se trabaja) En algunos pases socialistas y en alg un
momento de su historia se estableci o una ley laboral por la que en las factoras e instituciones
27
28 CAP

ITULO 3. ESPERANZA
estatales se trabajaba todos los das de la semana. La excepci on a esta regla era que algunos
de los trabajadores cumpliera a nos, ese da nadie trabajaba. Suponiendo un a no no bisiesto y
que los cumplea nos se distribuyen equiprobablemente a lo largo del a no, encontrar el n umero de
trabajadores para que el n umero esperado de hombres-da trabajados sea m aximo.
Problema 136 Elegimos al azar un n umero del 1 al 10. Hemos de adivinarlo mediante pre-
guntas cuya respuesta sea s o no. Calcular el n umero esperado de preguntas que debemos hacer
si,
1. la pregunta i-esima que hacemos es se trata del n umero i?, o
2. con cada pregunta intentamos eliminar la mitad de los n umeros restantes.
Problema 137 Un vendedor de peri odicos compra cada ejemplar a 10 centimos y los vende
a 15 centimos. Como no puede devolver los ejemplares no vendidos, trata de saber cuantos le
conviene comprar para maximizar lo que espera ganar cada da. Si las demandas diarias siguen
una distribuci on B(10, 1/3), cu antos ejemplares debe comprar?
Problema 138 Un vendedor de peri odicos compra cada ejemplar a C centimos de euro y
lo vende a V centimos de euro (C < V ). Puede devolver los ejemplares no vendidos pero s olo
obtiene R centimos de euro por ejemplar (R < C). Se trata de saber cuantos le conviene comprar
para maximizar lo que espera ganar cada da. Si la demanda diaria X sigue una distribuci on
discreta con P(X = k) = p
k
, k = 0, 1, 2, . . . , n, cu antos ejemplares debe comprar? Aplicar el
resultado para el caso en que V = 100, C = 50, R = 20 y la funci on de probabilidad viene dada
por
k P(X=k) k P(X=k)
0 0.05 7 0.08
1 0.08 8 0.05
2 0.10 9 0.04
3 0.15 10 0.03
4 0.15 11 0.03
5 0.10 12 0.03
6 0.08 13 0.03
Problema 139 Si X es una variable aleatoria continua con media 0 y varianza
2
y funci on
de distribuci on F, comprobar que se verica
F(x)
x
2
x
2
+
2
, si x > 0 y F(x)

2
x
2
+
2
, si x < 0.
Problema 140 Un juego consiste en lanzar un dado repetidas veces. El juego se detiene bien
porque parece un 1, bien porque decidimos detenerlo en cualquier lanzamiento antes de que haya
aparecido el n umero 1. El premio que recibimos es el n umero de puntos que muestra la cara del
dado en el ultimo lanzamiento. Cu al es la estrategia m as conveniente para obtener el mayor
premio?
Problema 141 Sea X una variable aleatoria no negativa con funci on de densidad f. Demos-
trar que
E(X
r
) =
_

0
rx
r1
P(X > x)dx.
3.1. ESPERANZA DE UNA VARIABLE ALEATORIA 29
Problema 142 Sea X una variable aleatoria continua con media y funci on de distribuci on
F. Demostrar que
_

F(x)dx =
_

(1 F(x))dx.
Problema 143 Una caja contiene a bolas blancas y b bolas negras, con a > 0 y b > 0. Se
extrae una bola al azar y si es negra el proceso termina. Si la bola es blanca se reintegra a la
caja junto con otra bola blanca y el proceso continua. Calcular la probabilidad de que el proceso
termine en la k-esima extracci on. Calcular la probabilidad de que el n umero de extracciones sea
nito. Obtener el n umero esperado de extracciones.
Problema 144 (Un camino aleatorio en la recta) Un punto se desplaza sobre el eje x a
derecha o izquierda de unidad en unidad. La probabilidad de que se desplace a la derecha es p
y 1 p de que lo haga a la izquierda. Si la variable X representa la posici on del punto despues
de n desplazamientos, partiendo de 0, obtener su distribuci on de probabilidad y calcular E(X)
y var(X).
Problema 145 (N umero esperado de parejas restantes) Se trata de un problema pro-
puesto y resuelto por Daniel Bernoulli en el siglo XVIII. Se propona averiguar el n umero
esperado de parejas que permanecen completas, de un total de N parejas iniciales, despues de
la muerte de m sus miembros.
Problema 146 Sea X una variable aleatoria discreta con soporte D
X
= {x
1
, x
2
, . . . , x
m
}.
Calcular
lm
n
n
_
E(X
n
).
Problema 147 Se repite indenida e independientemente un experimento cuyo resultado es
E (exito), con probabilidad p, o F (fracaso), con probabilidad q = 1 p. Denotamos por
n
el resultado de la n-esima repetici on. Sea T la variable que designa el n umero mnimo de
repeticiones necesarias para alcanzar r exitos consecutivos. Demostrar que P(T = ) = 0 y
calcular E(T).
Problema 148 Si g(x) es una funci on creciente no negativa de x, probar que
P(X a)
E(g(X))
g(a)
.
Problema 149 Sea X una variable aleatoria con soporte D
X
= {0, 1, . . . , n} tal que E(X) =
var(X) = 1. Demostrar que para cualquier natural k,
P(X k + 1)
1
k
2
.
x
Problema 150 Tenemos 10 pares de zapatos y elegimos al azar 4 zapatos. Cu al es la proba-
bilidad de que no hayamos elegido ning un par? Si X es una variable aleatoria que representa el
n umero de pares elegidos, obtener el n umero medio de pares entre los 4 zapatos elegidos.
Problema 151 La funci on de densidad de la variable aleatoria X viene dada por
f(x) =
_

_
1
2
+cx
3
, x ] 1, 1[;
0, en el resto.
30 CAP

ITULO 3. ESPERANZA
1. Determinar los valores de c para que f(x) sea una densidad.
2. Calcular, si existen, los momentos de orden 1 y 2 de la variable Y = |X|

1
2
.
Problema 152 Denamos la funci on
f(x) =
_
ax
(s+1)
, si x > r;
0, si x r,
con r, s > 0.
1. Determinar a para que f(x) sea una funci on de densidad de probabilidad.
2. Si la variable aleatoria X tiene por densidad f(x), para qu`e valores de s existir a su
esperanza?
Problema 153 Un autob us tiene en su recorrido 15 paradas. Supongamos que en la primera
parada suben 20 personas. Cada una de ellas elige al azar e independientemente de las otras en
cu al de las 14 paradas restantes quiere bajar.
1. Si X
i
es una variable aleatoria que vale 1 si alguna de las personas baja en la parada i y
0 en caso contrario, calcular su distribuci on de probabilidad.
2. Calcular el n umero medio de paradas que debe realizar el autob us para que bajen todos los
pasajeros.
Problema 154 Elegimos un punto uniformemente en el crculo centrado en cero y de radio
uno. Supongamos que denotamos por la variable aleatoria que nos da el angulo aleatorio
asociado a este punto y por X la variable aleatoria que nos da la abscisa del punto. Se pide:
1. Cu al es la distribuci on de ? Hay que obtener tanto la funci on de densidad como la
funci on de distribuci on.
2. Determinar P( [a, b]) para cualesquiera a y b con 0 a b 2.
3. Determinar la funci on de distribuci on de la variable X.
4. Determinar la funci on de densidad de la variable X.
5. Calcular E(X).
6. Calcular E(|X|).
Problema 155 Sea X una variable aleatoria con funci on de densidad
f(x) =
_
0, si x 0,
e
x
x
n
n!
, si x > 0,
donde n es un natural. Demostrar que se verica la siguiente desigualdad
P(0 < X < 2(n + 1)) >
n
n + 1
.
Problema 156 En una la de 15 butacas de un cine se sientan aleatoriamente 7 mujeres y
8 hombres. Por termino medio, cuantas parejas de asientos adyacentes est an ocupadas por
personas de distinto sexo.
3.1. ESPERANZA DE UNA VARIABLE ALEATORIA 31
Problema 157 Un reba no de ovejas es sometido a examen para detectar aquellas que padecen
determinada enfermedad. Sabemos que cada una de ellas la padece con probabilidad p e inde-
pendientemente de las otras. La detecci on de la enfermedad requiere un an alisis de sangre y
si se mezcla la sangre de n ovejas, basta que una de ellas padezca la enfermedad para que el
an alisis de positivo. Como el reba no es grande se plantean dos posibles estrategias:
1. examinar los animales uno a uno y llevar a cabo tantos an alisis como animales tiene el
reba no, o
2. examinar a los animales por grupos de n elementos, si el an alisis es negativo todos los del
grupo est an sanos, pero si es positivo habr a que analizar uno a uno todos los animales del
grupo.
Determinar cu al de las dos estrategias es mejor porque conduce a un n umero menor de an alisis.
Problema 158 Una variable aleatoria toma valores enteros positivos con probabilidades decre-
ciendo en progresi on geometrica. Elegir el primer termino y la raz on de la progresi on para que
E(X) = 10, y calcular, bajo dichas condiciones, P(X 10).
Problema 159 Para simular una moneda correcta a partir de una sesgada en la que la proba-
bilidad de obtener una cara vale = 1/2 podemos proceder como sigue. Lanzamos dos veces la
moneda e interpretamos C+ como cara y +C como cruz. Si no aparece ninguno de estos dos
resultados, repetimos los pares de lanzamientos hasta que aparezca alguno de ellos. Encontrar la
distribuci on de probabilidad y la esperanza del n umero de pares de lanzamientos necesarios para
poder tomar una decisi on (cara o cruz) y comprobar que, en efecto, se trata de la simulaci on
de una moneda correcta.
Problema 160 Determinar el valor esperado de la variable X denida en el problema 110.
Problema 161 Sea X una variable aleatoria no negativa. Demostrar que [E(X)]
1/2
E(X
1/2
).
Problema 162 Sea X el n umero de pruebas de Bernoulli necesarias para obtener un exito y
un fracaso. Determinar la distribuci on de probabilidad de X y calcular E(X) y var(X).
Problema 163 Determinar la media y la varianza de la variable X denida en el problema
83.
Problema 164 (Examen 3-2-2004) Un jugador puede apostar a cualquiera de los n umeros
enteros entre 1 y 6. Entonces lanza 3 dados y si aparece el n umero que eligi o, recibe como
premio su apuesta multiplicada por el n umero de dados que lo muestran y adem as le devuelven
lo que apost o. En otro caso pierde su dinero. Cu al es la ganancia esperada de este juego?
Problema 165 (Examen 8-6-2004) La variable aleatoria X, que toma valores en el inter-
valo [0,2], tiene por densidad la recta que pasa por (2,0) con pendiente negativa. Obtener su
funci on de densidad f(x) y calcular P(|X E(X)| 1/2). Que cota obtendramos para esta
probabilidad si utiliz aramos la desigualdad de Chebychev?
Problema 166 (Examen 8-6-2004) Tenemos una urna con 12 bolas numeradas de 1 a 12.
Extraemos dos bolas y denotamos por X
1
y X
2
los valores que observamos en la primera y en
la segunda extracci on. Sea X la variable denida como el m aximo de las dos extracciones. Se
pide:
1. La funci on de distribuci on de la variable X si suponemos que las dos extracciones se
realizan con reemplazamiento.
32 CAP

ITULO 3. ESPERANZA
2. La funci on de distribuci on de la variable X si suponemos que no hay reemplazamiento
entre las extracciones sucesivas.
3. La media de la variable X en las dos situaciones anteriores.
Problema 167 (Examen 1-9-2004) A partir de las variables aleatorias U
1
, . . . , U
n
, indepen-
dientes y todas ellas con distribuci on uniforme en el intervalo [0,1], se obtienen las variables
Y
i
= logU
i
, para i = 1, , n.
a) Que distribuci on siguen las variables Y
1
, , Y
n
? Son independientes?
b) Sea

Y =
1
n

n
i=1
Y
i
. Calcular E(Y ), V ar(Y ) y demostrar que,
k > 0, lm
n
P(|Y 1| > k) = 0.
c) Que distribuci on de probabilidad sigue Y ?
Problema 168 Determinar la media y varianza de la variable X denida en el problema 75.
3.2. Esperanza de un vector aleatorio
Problema 169 Las variables aleatorias X e Y tienen media 0, varianza 1 y su coeciente
de correlaci on es , Encontrar la media y la varianza de Z = X Y y sus coecientes de
correlaci on con X e Y .
Problema 170 Las variables aleatorias X
1
, X
2
, . . . , X
m+n
, n > m, son independientes e identi-
camente distribuidas con varianza com un,
2
, nita. Calcular el coeciente de correlaci on de
U =

n
i=1
X
i
y V =

m+n
j=m+1
X
j
.
Problema 171 Una urna contiene 2
n
bolas numeradas de 1 a n, de forma que hay
_
n
i
_
bolas
con el n umero i. Si extraemos m sin reemplazamiento y por S denotamos la suma de sus
n umeros, calcular E(S) y var(S).
Problema 172 (Un camino aleatorio en el plano) Un punto se desplaza sobre el plano
con movimientos independientes de manera que en cada movimiento la distancia que recorre es
siempre la misma, por ejemplo una unidad, pero la direcci on es aleatoria determinada por un
angulo cuya orientaci on respecto la posici on del punto se elige al azar en el intervalo [0, 2]. Si
D es la distancia del punto a su posici on original despues de n movimientos, calcular E(D
2
).
Problema 173 (N umero esperado de coincidencias) Recordemos que el llamado proble-
ma de las coincidencias consiste, en una de sus m ultiples versiones, en n individuos que al
abandonar una esta recogen sus sombreros al azar. Si por X denotamos el n umero de indivi-
duos que han cogido su propio sombrero (coincidencias), calcular E(X) y var(X).
Problema 174 Se realizan n pruebas independientes de un experimento que tiene k resultados
posibles A
1
, A
2
, . . . , A
k
con probabilidades p
1
, p
2
, . . . , p
k
,

p
i
= 1. Sea Y la variable n umero
de resultados que no han aparecido en ninguna de las n pruebas.
1. Que distribuci on tiene cada una de las variables X
j
que indica el n umero de veces que
ha sucedido A
j
?
2. Cu al es el valor esperado de Y ?
3.2. ESPERANZA DE UN VECTOR ALEATORIO 33
3. Comprobar que los valores de p
1
, . . . , p
k
que hacen mnima E(Y ) son p
1
= p
2
= =
p
k
= 1/k.
Problema 175 Sean X e Y dos variables aleatorias identicamente distribuidas, obtener cov(X+
Y, X Y ).
Problema 176 Sean X
1
, X
2
, . . . una familia de variables aleatorias independientes con la mis-
ma media y la misma varianza
2
, y sea Y
n
= X
n
+ X
n+1
+ X
n+2
. Hallar, para j 0,
cov(Y
n
, Y
n+j
).
Problema 177 Sea X
1
, X
2
, . . . una sucesi on de variables aleatorias continuas independientes e
identicamente distribuidas. Sea N 2 el ndice correspondiente a la variable donde la sucesi on
deja de ser decreciente, es decir, X
1
X
2
. . . X
N1
< X
N
. Demostrar que E(N) = e.
Sugerencia.- Obtener primero P(N n).
Problema 178 Sean X e Y el n umero de exitos y fracasos, respectivamente, cuando llevamos
a cabo n pruebas Bernoulli con probabilidad de exito p. Calcular el coeciente de correlaci on
entre ambas,
XY
.
Problema 179 Lanzamos tres veces consecutivas una moneda y denimos las variables alea-
torias X ={n umero de caras en los dos primeros lanzamientos} e Y ={n umero de caras en
los dos ultimos lanzamientos}. Obtener la distribuci on de probabilidad conjunta de X e Y , sus
marginales y el coeciente de correlaci on entre ambas.
Problema 180 Obtener el coeciente de correlaci on del vector (X, Y ) denido en el problema
94.
Problema 181 ( ) Sean X e Y dos variables aleatorias indicatrices independientes tales que
P(X = 1) = p y P(Y = 1) = q. Halla E[(X Y )
2
] en terminos de p y q.
Problema 182 ( ) La densidad conjunta de las variables X e Y es
f
XY
(x, y) =
_
c(y
2
x
2
)e
y
si y < x < y , y > 0,
0 en el resto,
donde c es una constante.
1. Demuestra que Y sigue una distribuci on gamma y deduce que c =
1
8
.
2. Halla la densidad de 4Y
3
.
3. Obten las esperanzas y varianzas de X e Y .
Problema 183 (Un ejemplo de fabricaci on simple) Un fabricante de bolas de golf est a eva-
luando un nuevo sistema de producci on. En este nuevo proceso de fabricaci on la probabilidad
de una bola defectuosa, que no puede ser vendida, es de 0.05 mientras que en el procedimiento
que actualmente utiliza es de 0.08. Con el procedimiento que utiliza el coste de producci on de
cada unidad es de 40 pesetas por bola mientras que en el nuevo el coste de cada bola es de 60
pesetas. Las bolas se venden a 125 pesetas cada una. Si el fabricante desea tener una ganancia
esperada m axima, que procedimiento de fabricaci on ha de utilizar?
Problema 184 (Variables discretas simetricas) Supongamos X una variable aleatoria dis-
creta tal que P(X = a x) = P(X = a + x) para cualquier x. Determinar la esperanza de la
variable X.
34 CAP

ITULO 3. ESPERANZA
Problema 185 (Examen 11-2-2000) Sean U
1
y U
2
dos variables aletorias independientes
ambas uniformes en [0,1]. Denimos X =

U
1
e Y = 2XU
2
.
1. Obtener la densidad conjunta de (X, Y )
2. Obtener las densidades marginales de X e Y y sus esperanzas.
3. Si W es una variable Bernoulli tal que P(W = 1) = P(0 Y

X), calcular E(W).


Problema 186 ( ) El peso de un tumor tras un tiempo t, W
t
, viene dado por la f ormula
W
t
= Xe
tY
donde X e Y son variables aleatorias independientes, X sigue una distribuci on
gamma con media 2 y varianza 1 e Y est a uniformemente distribuida en el intervalo (1, 1,5).
Halla E(W
t
) y var(W
t
).
Problema 187 (Krief y Levy, pagina 221) Sean X, Y y Z tres variables aleatorias. Se
supone que las tres parejas (X, Y ), (Y, Z) y (Z, X) tienen el mismo coeciente de correlaci on
r. Probar que se verica
r
1
2
. (3.1)
Se podr a, en un primer momento, suponer que las variables aleatorias X, Y y Z tienen la
misma varianza.
Problema 188 (Krief y Levy, pagina 223) Sea X una variable aleatoria continua que ad-
mite una funci on de densidad f(x) y que tiene momentos nitos de los dos primeros ordenes.
Se supone que la funci on f es par, es decir, que se tiene que para todo x,
f(x) = f(x). (3.2)
Probar que:
1. Probar que E(X) = 0.
2. Probar que el coeciente de correlaci on r entre X y |X| es nulo. Conclusi on?
Problema 189 ( ) Un terremoto de magnitud M libera una cantidad de energa X tal que
M = lnX. Para terremotos de magnitud mayor que 3 sup on que M 3 tiene una distribuci on
exponencial de media 2.
1. Halla E(M) y V ar(M) para un terremoto de magnitud mayor que 3
2. Para un terremoto como el del apartado anterior halla la densidad de X
3. Consideremos dos terremotos, que han sucedido independientemente, ambos de magnitud
mayor que 3. Cu al es la probabilidad de que la magnitud del terremoto m as peque no sea
mayor de 4?
Problema 190 Un accidente tiene lugar en un punto X U(0, L). En el instante del accidente
una ambulancia est a en una localizaci on Y tambien distribuida uniformemente en la carretera
de longitud L. Halla la distancia esperada E(| X Y |) entre el accidente y la ambulancia.
Problema 191 (Uno de ascensores) Un edicio tiene 10 pisos. 12 personas suben a un as-
censor y cada una de ellas elige aleatoriamente el piso en donde quiere bajar (es decir, elige con
la misma probabilidad cada uno de los posibles pisos). Adem as hace su elecci on independiente-
mente de los dem as. Cu al es el n umero medio de pisos en los que ha de parar el ascensor con
objeto de que baje una o m as personas?
3.3. ESPERANZA CONDICIONADA 35
3.3. Esperanza condicionada
Problema 192 Una m aquina fabrica bras de longitud aleatoria X. Para medir la desigualdad
entre las longitudes de las bras fabricadas se utiliza el coeciente
=
a

a
,
donde a = E(X), a

= E(X | X > a) y a

= E(X | X < a). Si X N(a,


2
), encontrar la
relaci on entre , a y .
Problema 193 (Un problema de pastillas. La distribuci on Hipergeometrica negativa)
Una caja contiene pastillas de dos tipos: grandes y peque nas. Cada pastilla grande equivale a
dos peque nas. Cada da el paciente debe tomar una de las pastillas, que elige al azar. Si es
de las grandes la parte en dos, toma una mitad y devuelve la otra a la caja. Si la pastilla es
peque na la toma sin m as. A todos los efectos las mitades devueltas a la caja son consideradas
como pastillas peque nas. Cu al es el n umero esperado de pastillas peque nas que quedar an en la
caja cuando las grandes se hayan terminado?
Problema 194 El n umero de clientes en la cola de la caja de un supermercado sigue una
distribuci on de Poisson con par ametro . El tiempo que tarda cada cliente en ser atendido
sigue una Exponencial de par ametro . Calcular el tiempo medio que debemos esperar en la
cola.
Problema 195 El n umero de pasajeros que espera el tren en cierta estaci on en un instante t
es una variable aleatoria Poisson de par ametro t. Si el tiempo de llegada del tren se distribuye
uniformemente en el intervalo [0, T], cu al es el n umero medio de pasajeros que subir a al tren?
Obtener tambien su varianza.
Problema 196 Un minero atrapado en una mina tiene tres posibles caminos para tratar de
escapar de la mina. El primero le conduce al exterior despues de 3 horas. El segundo le conduce
de nuevo al interior de la mina despues de un recorrido de 5 horas. El tercero le conduce tambien
al interior de la mina, pero despues de 7 horas de recorrido. Si el minero echa a suertes (igual
probabilidad) el camino por el que tratar de escapar, cu al ser a el tiempo de medio que tardar a en
conseguirlo?
Problema 197 Sea {X
j
}
j1
una sucesi on de variables aleatorias uniformes en el intervalo
[0, 1]. Calcular E(N) siendo
N = mn
_
_
_
n;
n

j=1
X
j
> 1
_
_
_
.
Problema 198 El vector aleatorio (X, Y ) tiene por densidad conjunta
f
XY
(x, y) =
_

_
1
2
ye
yx
, 1 < y < 3, x > 0;
0, en el resto.
Hallar E(X) y var(X) haciendo uso de E(X|Y ) y de var(X|Y ).
36 CAP

ITULO 3. ESPERANZA
Problema 199 El vector aleatorio (X, Y ) tiene por densidad conjunta
f
XY
(x, y) =
_

_
2e
2x
x
, 0 x < +, 0 y x;
0, en el resto.
Calcular cov(X, Y ).
Sugerencia.- La obtenci on de E(Y ) puede resultar m as sencilla a traves de E(Y ) = E[E(Y |X)].
Problema 200 Tenemos un lote de N pastillas y antes de comercializarlo lo sometemos al
siguiente control de calidad. Fijamos un umbral c (c < n) y a continuaci on tomamos una
muestra de n pastillas del total de N que componen el lote. Sea X la variable aleatoria que nos
da el n umero de pastillas en mal estado en la muestra, si X c entonces sustituimos las X
pastillas defectuosas por pastillas en buen estado y comercializamos, sin m as inspecci on, el lote.
En el caso en que X > c entonces se muestrean todas y cada una de las pastillas que componen
el lote y se sustituyen por pastillas en buen estado, comercializ andolo a continuaci on. En este
caso el lote no tiene ninguna pastilla en mal estado. Si la probabilidad de que el proceso de
fabricaci on produzca una pastilla en mal estado es p, que n umero medio de pastillas en mal
estado estamos lanzando al mercado?
Problema 201 Elegimos al azar dos puntos en el intervalo [0, a]. Queremos conocer,
a) Area media del rect angulo que tiene por lados las correspondientes longitudes.
b) Area media de dicho rect angulo si ambos puntos son mayores que a/2.
Problema 202 Un trabajador est a encargado del correcto funcionamiento de n m aquinas si-
tuadas en linea recta y distantes una de otra l metros. El trabajador debe repararlas cuando se
averan, cosa que sucede con igual probabilidad para todas ellas e independientemente de una a
otra. El operario puede seguir dos estrategias:
1. acudir a reparar la m aquina estropeada y permanecer en ella hasta que otra m aquina se
avera, desplaz andose entonces hacia ella, o
2. situarse en el punto medio de la linea de m aquinas y desde all acudir a la averiada,
regresando nuevamente a dicho punto cuando la avera est a resuelta.
Si X es la distancia que recorre el trabajador entre dos averas consecutivas, c ual de ambas
estrategias le conviene m as para andar menos?
Problema 203 Sean X e Y dos variables aleatorias. Si E(X|Y ) = 10Y y E(Y |X) = 7X/4,
obtener su coeciente de correlaci on.
Problema 204 Se pide determinar la media y la varianza de la variable X denida en el
problema 111.
Problema 205 Calcular el coeciente de correlaci on entre el mayor y el menor valor obtenidos
al lanzar dos dados.
Problema 206 Obtener el coeciente de correlaci on entre las coordenadas de un punto elegido
al azar en el crculo unidad.
3.3. ESPERANZA CONDICIONADA 37
Problema 207 (Examen 8-6-2004) La variable aleatoria X se distribuye exponencialmente
con par ametro Y , que es a su vez una variable aleatoria uniforme en [1, 4]. Obtener la distri-
buci on conjunta de X e Y y la esperanza y la varianza de X.
Problema 208 (Examen 1-9-2004) Sea X una variable aleatoria con distribuci on normal
est andar (con media 0 y varianza 1) y sea I otra variable aleatoria, independiente de X y tal
que P(I = 1) = P(I = 0) = 1/2. Se dene la variable aleatoria Y mediante
Y =
_
X, si I = 1;
X, si I = 0.
1. Calcular las probabilidades P(X < 1, Y > 1), P(X < 1) y P(Y > 1). Son independientes
X e Y ? Justica la respuesta.
2. Demostrar que Y sigue una distribuci on normal est andar.
3. Calcular E[XY |I = 1] y E[XY |I = 0] y demostrar que Cov(X, Y ) = 0.
Problema 209 (Examen 21-6-2005) Sobre un crculo cuyo radio R es aleatorio con funci on
de densidad
f
R
(r) =
_

_
r
2
9
, r [0, 3];
0 en el resto,
elegimos un punto al azar. Si X designa la distancia del punto al origen, obtener
1. La funci on de distribuci on y la funci on de densidad de X|R = r.
2. La media y la varianza de X.
38 CAP

ITULO 3. ESPERANZA
Captulo 4
Convergencia de sucesiones de
variables aleatorias
4.1. Tipos de convergencia
Problema 210 Consideremos la variable aleatoria X U(0, 1) y denamos la sucesi on X
n
=
X/n, n 1. Comprobar que X
n
L
Y , siendo Y una variable aleatoria degenerada, P(Y = 0) =
1.
Problema 211 La sucesi on de variables aleatorias {X
n
}
n1
tales que P(X
n
= 1 1/n) =
P(X
n
= 1 + 1/n) = 1/2, convergen en ley a la variable aleatoria X tal que P(X = 1) = 1.
Tienden sus funciones de probabilidad a una funci on de probabilidad?
Problema 212 Sean X
j
, j = 1, . . . , n, variables aleatorias independientes, todas ellas U(0, 1).
Denimos
Y
n
= mn(X
1
, . . . , X
n
), Z
n
= max(X
1
, . . . , X
n
), U
n
= nY
n
, V
n
= n(1 Z
n
).
Demostrar que cuando n +,
Y
n
P
0; Z
n
P
1; U
n
L
U; V
n
P
V,
siendo U y V variables aleatorias exponenciales con par ametro = 1.
Problema 213 Aplicar el anterior resultado a las variables aleatorias X
j
, j = 1, . . . , n, inde-
pendientes dos a dos y tales que
P(X
j
= a
j
) = P(X
j
= a
j
) =
1
2
.
Para que valores de a es aplicable el resultado?
Problema 214 Consideremos x [0, 1] y sea
n
(x)={el n-esimo dgito de su expresi on deci-
mal}. Denamos S
n
(x) =

n
k=1

k
(x) y
A
n
(y) =
_
x;
2S
n
(x) 9n

33n
< y
_
,
39
40 CAP

ITULO 4. CONVERGENCIA DE SUCESIONES DE VARIABLES ALEATORIAS


y sea (A
n
(y)) la medida de Lebesgue de A
n
(y). Probar que
lm
n
(A
n
(y)) =
1

2
_
y

u
2
2
du.
Problema 215 La variable aleatoria Y se distribuye Exp(1). Denimos
X
n
=
_
1, si Y < lnn;
0, en caso contario.
Obtener la funci on de distribuci on de X
n
y estudiar la convergencia en ley de las X
n
.
Problema 216 Sea T U(1/c, 1/c), c > 0, y denamos Y = cT. Para k N, las variables
aleatorias X
k
se denen mediante,
X
k
=
_
_
_
1, si 1 < Y < 1/k;
0, si 1/k Y < 1/k;
1, si 1/k Y < 1.
Demostrar que X
k
converge en ley a la variable aleatoria X con funci on de probabilidad f
X
(1) =
f
X
(1) = 1/2 y f
X
(x) = 0, x / {1, 1}.
Problema 217 (Examen 1-9-2004) Una empresa que alquila coches con conductor ha ob-
servado que el n umero de kil ometros por da de alquiler que se hacen con un determinado tipo
de vehculo sigue una distribuci on N(200, 5625).
a) Cu al es la probabilidad de que en 30 das se hagan m as de 5.000 kil ometros? Que hip ote-
sis hay que asumir?
b) La compa na revisa el estado del vehculo si en un mismo da hace m as de 350 km. Calcular
el n umero esperado de das que la compa na puede alquilar el vehculo antes de tener que
revisarlo.
4.2. Leyes de los Grandes N umeros
Problema 218 Demostrar que la independencia de las variables en la ley debil de los grandes
n umeros puede relajarse exigiendo solamente independencia dos a dos y acotaci on de las va-
rianzas. Es decir, si X
j
, j = 1, . . . , n, verican que E(X
j
) =
j
y var(X
j
) =
2
j
son nitas,
entonces
X
n

n
P
0,
donde

n
=
1
n
n

j=1

j
, X
n
=
1
n
n

j=1
X
j
las X
j
son independientes dos a dos y
2
j
M, j.
Problema 219 Sea S
n
el n umero de exitos en n pruebas Bernoulli con probabilidad de exito
p en cada prueba. Encontrar una cota para P(|S
n
/n p| ) que no dependa de p.
Problema 220 Tenemos dos monedas, una correcta y otra con probabilidad de cara p = 3/4.
Elegimos al azar una de las dos y realizamos una serie de lanzamientos. Despues de observar el
resultado de un gran n umero de ellos, podemos saber la moneda elegida? Cu al es el mnimo
n umero de lanzamientos para poder saberlo con una probabilidad de al menos 95 %?
4.3. FUNCI

ON CARACTER

ISTICA 41
4.3. Funcion caracterstica
Problema 221 Probar que una variable aleatoria X es simetrica s y s olo s su funci on carac-
terstica es real.
Problema 222 Encontrar la distribuci on de las variables aleatorias que tiene por funci on ca-
racterstica
1)
1
(t) =

k0
a
k
cos kt, 2)
2
(t) =

k0
a
k
e
i
k
t
.
4.4. Teorema Central de Lmite
Problema 223 Ante la caja de un banco hay 60 personas que esperan recibir su salario, del
que sabemos que es una cantidad aleatoria de media = 100 euros y desviaci on tpica = 30
euros. Si los salarios son independientes de un asalariado a otro, queremos saber:
1. Cu anto dinero debe tener la caja para, con probabilidad 0,95, poder pagar todos los sala-
rios?
2. si la caja cuenta incialmente con 7,000 euros, cu al es la probabilidad de que al nal de
los pagos queden en caja al menos 500 euros?
Problema 224 Una empresa produce 10000 bolas de acero para rodamientos, siendo p = 0,05
la probabilidad de que una bola sea defectuosa. El proceso de producci on garantiza que las bolas
son fabricadas independientemente unas de otras. Las bolas defectuosas son arrojadas a un
recipiente cuya capacidad queremos determinar para que, con probabilidad 0.99, quepan en el
todas las bolas defectuosas del proceso.
Problema 225 Las bombillas utilizadas por cierto aparato pueden ser de dos clases, A y B.
Las de la clase A tienen una duraci on media
A
= 2000 horas con devsiaci on tpica
A
= 400
horas, mientras que las de la clase B tienen una duraci on media
B
= 1800 horas con desviaci on
tpica
B
= 500 horas. Se compran 200 bombillas de la clase A y 150 de la clase B. Calcular
la probabilidad de que la duraci on media de la muestra de la clase A no supere en m as de 100
horas la duraci on media de la muestra de la clase B.
Problema 226 Un colegio est a preparando la esta de graduaci on de sus 500 estudiantes. Se
sabe, por lo ocurrido en otras ocasiones, que el 50 % de los estudiantes vienen acompa nados de
sus padres, el 30 % s olo por uno de ellos y el 20 % restante vienen solos. Cuantos asientos hay
que disponer para los padres si queremos que, con una probabilidad superior a 0.95, todos ellos
puedan sentarse?
Problema 227 Disponemos de un dado cargado en el que la probabilidad de obtener cualquiera
de las caras es proporcional a su n umero de puntos. Jugamos con el pagando 4 euros por jugada
y recibiendo como premio tantos euros como puntos tiene la cara obtenida al lanzarlo. Obtener
la probabilidad aproximada de ir ganando al cabo de 100 jugadas.
Problema 228 Probar con argumentos probabilsticos que
e
n
n

k=0
n
k
k!
n

1
2
.
42 CAP

ITULO 4. CONVERGENCIA DE SUCESIONES DE VARIABLES ALEATORIAS


Problema 229 La fabricaci on de zumo de naranja se lleva a cabo por lotes de n envases. La
caducidad en das de cada envase es una variable aleatoria de media = 20 y varianza
2
= 9.
Obtener la media y la varianza de la caducidad media del lote y calcular el tama no mnimo del
lote para que con probabilidad 0,99 dicha caducidad media supere los 19 das.
Problema 230 (Todava existen las pesetas) Un programa de contabilidad redondea las
cantidades (expresadas en pesetas con una precisi on de dos decimales) a la peseta m as pr oxima
de manera que las cantidades con centimos entre 0 y 49 quitamos los centimos correspondientes
mientras que las cantidades con centimos entre 50 y 99 lo redondeamos a la unidad superior.
Determinar la probabilidad aproximada que que el error acumulado en 100 transacciones sea
mayor que 5 pesetas (por exceso o por defecto) suponiendo que el n umero de centimos que nos
aparece en una cantidad cualquiera se distribuye uniformemente en {0, . . . , 99}.
Problema 231 Dos compa nas aereas A y B ofrecen el mismo servicio en dos vuelos diferentes
que salen al mismo tiempo (en denitiva, es igualmente probable que un pasajero coja uno de los
vuelos). Supongamos que ambas compa nas compiten por un grupo de 400 pasajeros potenciales.
La compa na A vende un billete a cualquiera que se lo solicita y la capacidad de su avi on es
de 230 pasajeros. Determinar la probabilidad aproximada de que A tenga un overbooking (que
alg un pasajero tenga la desagradable sorpresa de tener un billete pero no tenga asiento).
Problema 232 (Examen 8-6-2004) Se redondean 20 n umeros al entero m as cercano y des-
pues se suman. Supongamos que los errores de redondeo son independientes y uniformemente
distribuidos en el intervalo [
1
2
,
1
2
]. Se pide determinar la probabilidad de que la suma obtenida
diera de la suma de los 20 n umeros originales en m as de 3 unidades.
4.5. Funcion generatriz de momentos
Problema 233 1. Halla la funci on generatriz de momentos de X Po().
2. Obten la esperanza y la varianza de X.
3. Demuestra que si X
i
Po(
i
) para i = 1, . . . , k y son variables independientes entonces
S =

k
i=1
X
i
Po(), siendo =

k
i=1

i
.
Problema 234 1. Demuestra que si X
i
Exp() para i = 1, . . . , k y son variables aleato-
rias independientes entonces S =

k
i=1
X
i
Ga(k, ).
2. Halla E(S) y V ar(S).
Problema 235 Sea X
1
, . . . X
n
una muestra aleatoria de una N(, ) demostrar que X
n

N(,

n
), siendo X
n
=
1
n

n
i=1
X
i
.
Problema 236 Si X
i

2
ni
para i = 1, . . . , k y son variables aleatorias independientes enton-
ces S =

k
i=1
X
i

2
n
, siendo n =

k
i=1
n
i
.
Problema 237 Halla la funci on generatriz de momentos de X BN(r, p) y calcular E(X).
Captulo 5
Examenes previos
5.1. 1 de septiembre de 2004
5.1.1. Castellano
Problema 238 Supongamos que una caja contiene 5 monedas, cada una de ellas con distin-
ta probabilidad de obtener una cara la lanzarla. Denotamos por p
i
dicha probabilidad para la
moneda i, i = 1, 2, . . . , 5 y supongamos que p
1
= 0, p
2
= 1/4, p
3
= 1/2, p
4
= 3/4 y p
5
= 1.
a) Se selecciona al azar una moneda. Si la primera cara se obtiene al cuarto lanzamiento,
cu al es la probabilidad de que la iesima moneda haya sido la seleccionada?
b) Si se lanza de nuevo la misma moneda, cu al es la probabilidad de obtener una nueva
cara?
c) Si se hubiera obtenido una cruz en el primer lanzamiento, cu al es la probabilidad de
obtener una cara en el segundo lanzamiento?
Problema 239 Sea X una variable aleatoria con distribuci on normal est andar (con media 0
y varianza 1) y sea I otra variable aleatoria, independiente de X y tal que P(I = 1) = P(I =
0) = 1/2. Se dene la variable aleatoria Y mediante
Y =
_
X, si I = 1;
X, si I = 0.
1. Calcular las probabilidades P(X < 1, Y > 1), P(X < 1) y P(Y > 1). Son independientes
X e Y ? Justica la respuesta.
2. Demostrar que Y sigue una distribuci on normal est andar.
3. Calcular E[XY |I = 1] y E[XY |I = 0] y demostrar que Cov(X, Y ) = 0.
Problema 240 Una empresa que alquila coches con conductor ha observado que el n umero
de kil ometros por da de alquiler que se hacen con un determinado tipo de vehculo sigue una
distribuci on N(200, = 75).
a) Cu al es la probabilidad de que en 30 das se hagan m as de 5.000 kil ometros? Que hip ote-
sis hay que asumir?
43
44 CAP

ITULO 5. EX

AMENES PREVIOS
b) La compa na revisa el estado del vehculo si en un mismo da hace m as de 350 km. Calcular
el n umero esperado de das que la compa na puede alquilar el vehculo antes de tener que
revisarlo.
Problema 241 A partir de las variables aleatorias U
1
, . . . , U
n
, independientes y todas ellas
con distribuci on uniforme en el intervalo [0,1], se obtienen las variables Y
i
= logU
i
, para
i = 1, , n.
a) Que distribuci on siguen las variables Y
1
, , Y
n
? Son independientes?
b) Sea

Y =
1
n

n
i=1
Y
i
. Calcular E(Y ), V ar(Y ) y demostrar que,
k > 0, lm
n
P(|Y 1| > k) = 0.
c) Que distribuci on de probabilidad sigue Y ?
5.1.2. Valenciano
Problema 242 Una caixa conte 5 monedes, cadascuna delles amb distinta probabilitat de
mostrar una cara en ser llancada. Si per p
i
denotem la probabilitat de cara per a la moneda i,
tenim p
1
= 0, p
2
= 1/4, p
3
= 1/2, p
4
= 3/4 y p
5
= 1.
a) triem una moneda a latzar. Si la primera cara apareix en el quart llancament, quina es
la probabilitat de que la i`esima moneda haja estat la seleccionada?
b) Si llancem una altra vegada la mateixa moneda, quina es la probabilitat dobtenir una
cara?
c) Si al primer llancament haguerem tret una creu, quina hagues estat la probabilitat dobtenir
una cara en el segon llancament?
Problema 243 Siga X una variable aleat` oria amb distribuci o normal est andar (mitjana 0 i
varianca 1) i siga I una altra variable aleat` oria, independent dX i tal que P(I = 1) = P(I =
0) = 1/2. Denim Y mitjancant
Y =
_
X, si I = 1;
X, si I = 0.
1. Calcular les probabilitats P(X < 1, Y > 1), P(X < 1) i P(Y > 1). S on independents X i
Y ? Justica la resposta.
2. Demostrar que Y segueix una distribuci o normal est andar.
3. Calcular E[XY |I = 1] i E[XY |I = 0] i demostrar que Cov(X, Y ) = 0.
Problema 244 Una empresa de lloguer de cotxes amb ch ofer ha observat que el n umero de
quil` ometres per dia de lloguer que fa un cert tipu de vehicle segueix una distribuci o N(200, =
75).
a) Quina es la probabilidad de que un daquests vehicles fa ca mes de 5.000 quil` ometres en
30 dies? Quina hip` otesi hi ha que assumir?
b) La companyia revisa lestat del vehicle si en un mateix dia fa mes de 350 qm. Calcular el
nombre esperat de dies que la compa na pot llogar el cotxe sense haver de revisar-lo .
5.2. 3 DE FEBRERO DE 2004 45
Problema 245 A partir de les variables aleat` ories U
1
, . . . , U
n
, independents i totes elles amb
distribuci o uniforme en linterval [0,1], obtenim les variables Y
i
= logU
i
, i = 1, , n.
a) Quina distribuci o segueixen les variables Y
1
, , Y
n
? S on independents?
b) Siga

Y =
1
n

n
i=1
Y
i
. Calcular E(Y ), V ar(Y ) i demostrar que,
k > 0, lm
n
P(|Y 1| > k) = 0.
c) Quina distribuci o de probabilitat segueix Y ?
5.2. 3 de febrero de 2004
5.2.1. Castellano
Problema 246 Un jugador puede apostar a cualquiera de los n umeros enteros entre 1 y 6.
Entonces lanza 3 dados y si aparece el n umero que eligi o, recibe como premio su apuesta mul-
tiplicada por el n umero de dados que lo muestran y adem as le devuelven lo que apost o. En otro
caso pierde su dinero. Cu al es la ganancia esperada de este juego?
Problema 247 Un taxi se ve involucrado en un accidente nocturno. En la ciudad hay dos
compa nas de taxis, los taxis Negros y los taxis Blancos. Se sabe que el 85 % de los taxis de
la ciudad son Negros y el 15 % restante son Blancos. Un testigo del accidente arma que el
taxi involucrado era Blanco y la abilidad de su testimonio es del 80 %, es decir, es capaz de
identicar correctamente el color del taxi el 80 % de las veces.
1. Calcula la probabilidad de que el taxi accidentado fuera el Blanco, dado que el testigo
arma que lo era, y comp arala con la respuesta del testigo.
2. Supongamos que el 100p % de los taxis son Blancos, con 0 p 1, y que la abilidad
del testigo contin ua siendo del 80 %. Estudia la sensibilidad a los datos de la respuesta
anterior viendo como vara esta en funci on de p. A partir de que valor de p la anterior
probabilidad supera 0.5?
3. El an alisis anterior puede completarse permitiendo que la abilidad del testigo sea variable,
100q %, con 0 q 1. Determina la relaci on que se debe dar entre p y q para que la
probabilidad pedida supere 0.5.
Problema 248 El holandes Christian Huygens public o en 1657 uno de primeros libros sobre
Probabilidad que se conocen, De Ratiociniis in Ludo Aleae (Del Razonamiento en los Juegos de
Azar), en el que planteaba una serie de problemas. El que se conoce como segundo problema
de Huygens lo enunciamos a continuaci on.
Tres jugadores A, B y C participan en el siguiente juego. Una urna contiene a
bolas blancas y b negras. Los jugadores, en el orden ABCABC . . ., extraen una bola
con reemplazamiento hasta que uno de ellos obtiene una bola blanca y gana.
Encontrar la probabilidad de ganar para cada jugador.
Problema 249 Elegimos un punto uniformemente en el crculo centrado en cero y de radio
uno. Supongamos que denotamos por la variable aleatoria que nos da el angulo aleatorio
asociado a este punto y por X la variable aleatoria que nos da la abscisa del punto. Se pide:
46 CAP

ITULO 5. EX

AMENES PREVIOS
1. Cu al es la distribuci on de ? Hay que obtener tanto la funci on de densidad como la
funci on de distribuci on.
2. Determinar P( [a, b]) para cualesquiera a y b con 0 a b 2.
3. Determinar la funci on de distribuci on de la variable X.
4. Determinar la funci on de densidad de la variable X.
5. Calcular E(X).
6. Calcular E(|X|).
5.2.2. Valenciano
Problema 250 Un jugador pot apostar a qualsevol dels enters entre 1 y 6, tots dos inclosos.
Triat el n umero llanca 3 daus, i si algun dells mostra el n umero que ha triat rep com a premi
tantes vegades el que va apostar com daus mostren el n umero, i a mes a mes li tornen laposta.
Quin es el guany esperat en aquest joc?
Problema 251 Un taxi te un accident nocturn en una ciutat on hi ha dues companyes de
taxis, els Negres i els Blancs. Sabem que el 85 % dels taxis s on Negres i la resta blancs. Un
testimoni de laccident, que es capa c didenticar correctament el color del taxi en el 80 % de
les ocasions, arma que el taxi era Blanc.
1. Calcula la probabilitat de que el taxi accidentat fos blanc, donat que el testimoni aix ho
arma, i compara-la amb el que diu el testimoni.
2. Suposem ara que el 100p % dels taxis s on Blancs, on 0 p 1, i que la abilitat del
testimoni continua essent del 80 %. Estudia la sensibilitat de la probabilitat obtinguda a
lapartat anterior mitjancant la seua variaci o en funci o de p. A partir de quin valor de p
aquesta probabilitat supera 0,5?
3. Lanterior an` alisi pot completar-se permetent que la abilitat del testimoni siga variable,
100q %, on 0 q 1. Determina la relaci o que deuen tenir p i q per a que lesmentada
probabilitat supere 0,5.
Problema 252 Lholandes Christian Huygens va publicar en 1657 un dels primers llibres sobre
Probabilitat, De Ratiociniis in Ludo Aleae (Del Raonament en els Jocs dAtzar), on proposava
un seguit de problemes. El conegut com a segon problema de Huygens lenunciem tot seguit.
Tres jugadors A, B i C participen en el seg uent joc. Una urna te a bolas blanques i
b negres. Els jugadors, en lordre ABCABC . . ., trauen una bola amb reempla cament
ns que un dells trau una bola blanca i guanya.
Trobar la probabilitat de guanyar per a cada jugador.
Problema 253 Triem un punt a latzar dintre del cercle unitat centrat en zero. Designem
mitjancant i X, respectivament, langle aleatori i labscissa associats al punt. Es demana:
1. La distribuci on de (cal obtenir tant la funci o de densitat com la funci o de distribuci o).
2. Determinar P( [a, b]), a, b amb 0 a b 2.
3. Determinar la funci o de distribuci o de la variable X.
5.3. 3 DE SEPTIEMBRE DE 2005 47
4. Determinar la funci o de densitat de la variable X.
5. Calcular E(X).
6. Calcular E(|X|).
5.3. 3 de septiembre de 2005
5.3.1. Castellano
Problema 254 Tenemos 10 pares de zapatos y elegimos al azar 4 zapatos. Cu al es la proba-
bilidad de que no hayamos elegido ning un par? Si X es una variable aleatoria que representa el
n umero de pares elegidos, obtener el n umero medio de pares entre los 4 zapatos elegidos.
Ayuda.- Para el c alculo de E(X) puede ayudar denir variables X
j
que valen 1 si el par j ha
sido escogido y 0 en caso contrario.
Problema 255 Cuando una corriente de I amperios pasa a traves de una resistencia de R
ohmios, la potencia generada viene dada por W = I
2
R vatios. Supongamos que I y R son
variables aleatorias independientes con densidades
f
I
(x) =
_
_
_
6x(1 x), si 0 x 1;
0, fuera.
y
f
R
(y) =
_
_
_
2y, si 0 y 1;
0, fuera.
Hallar la densidad de W.
Problema 256 La variable aleatoria Y se distribuye Exp(1). Denimos
X
n
=
_
_
_
1, si Y < lnn;
0, en caso contario.
Obtener la funci on de distribuci on de X
n
y estudiar la convergencia en ley de las X
n
.
Problema 257 La probabilidad de que un virus inform atico haya infectado nuestro ordenador
es 0, 1. Si el ordenador est a infectado, un sistema antivirus detecta la infecci on con proba-
bilidad x = 0, 95, mientras que en caso de no infecci on el sistema detecta falsas infecciones
con probabilidad y = 0, 03. Interesa que el sistema antivirus tenga un elevado valor predicti-
vo={probabilidad de que el ordenador este infectado cuando el antivirus detecta una infecci on}.
Calcularlo a partir de los datos anteriores. Si queremos aumentarlo, donde hemos de dirigir
nuestros esfuerzos, a aumentar x o a rebajar y?
5.3.2. Valenciano
Problema 258 Tenim 10 parelles de sabates i triem a latzar 4 sabates. Quina es la probabilitat
de no haver triat cap parella? Si X es la variable aleat` oria que representa el n umero de parelles
que hem triat, obtenir la mitjana del n umero de parelles entre les 4 sabates que hem triat.
Ajut.- Per al c` alcul dE(X) pot ajudar denir variables X
j
que valen 1 si el parell de sabates
j ha estat triat i 0 en cas contrari.
48 CAP

ITULO 5. EX

AMENES PREVIOS
Problema 259 Un corrent dintensitat I ampers en creuar una resist`encia de R ohms, genera
una pot`encia de W = I
2
R vats. Suposem que I i R s on variables aleat` ories independents amb
densitats
f
I
(x) =
_
_
_
6x(1 x), si 0 x 1;
0, fora.
i
f
R
(y) =
_
_
_
2y, si 0 y 1;
0, fora.
Trobar la densitat de W.
Problema 260 La variable aleat` oria Y te per distribuci o una Exp(1). Denim
X
n
=
_
_
_
1, si Y < lnn;
0, altrament.
Obtenir la funci o de distribuci o de X
n
i estudiar la converg`encia en llei de les X
n
.
Problema 261 La probabilitat de que un virus inform` atic infecte el nostre ordinador es 0, 1.
Si lordinador est` a infectat, un sistema antivirus ho detecta amb probabilitat p
I|D
= 0, 95,
mentre que en caso de no ho haja fet el sistema detecta falses infeccions amb probabilitat 0, 03.
Interesa que el sistema antivirus tinga un alt valor predictiu={probabilitat de que el ordinador
estiga infectat quan lantivirus ho detecta}. Calcular-lo a partir de les dades anteriors. Si volem
augmentar-lo, qu`e ens conve, augmentar x o rebaixar y?
5.4. 8 de junio de 2004
5.4.1. Castellano
Problema 262 La variable aleatoria X, que toma valores en el intervalo [0,2], tiene por den-
sidad la recta que pasa por (2,0) con pendiente negativa. Obtener su funci on de densidad f(x)
y calcular P(|XE(X)| 1/2). Que cota obtendramos para esta probabilidad si utiliz aramos
la desigualdad de Chebychev?
Problema 263 Supongamos una clase de n estudiantes. Uno de ellos conoce un rumor que
cuenta a uno de sus compa neros elegido al azar. A su vez este segundo estudiante vuelve a
contarlo a otro compa nero elegido al azar y distinto del que se lo ha contado. Este rumor sigue
propag andose del mismo modo. En cada ocasi on el estudiante lo cuenta a otro elegido al azar
entre los n del grupo, excluyendo a aquel que se lo cont o. Cu al es la probabilidad de contar la
historia k veces sin que se la cuenten dos veces al mismo individuo?
Sugerencia.- Denid los sucesos A
i
={la historia no se le repite de nuevo a alguien que ya
la conoce cuando se cuenta por i-esima vez}, i = 1, . . . , k. A partir de estos sucesos se puede
denir el suceso de interes.
Problema 264 La variable aleatoria X se distribuye exponencialmente con par ametro Y , que
es a su vez una variable aleatoria uniforme en [1, 4]. Obtener la distribuci on conjunta de X e
Y y la esperanza y la varianza de X.
5.4. 8 DE JUNIO DE 2004 49
Problema 265 Se redondean 20 n umeros al entero m as cercano y despues se suman. Suponga-
mos que los errores de redondeo son independientes y uniformemente distribuidos en el intervalo
[
1
2
,
1
2
]. Se pide determinar la probabilidad de que la suma obtenida diera de la suma de los
20 n umeros originales en m as de 3 unidades.
Problema 266 Consideremos el siguiente procedimiento:
1. Generamos U con distribuci on uniforme en [0, 1].
2. Tomamos Y =
1

ln(1 U) siendo ln el logaritmo neperiano y una constante positiva.


3. Tomamos X = [Y ] donde [Y ] es la parte entera por exceso de Y .
Se pide:
1. La distribuci on de probabilidad de la variable aleatoria Y .
2. La funci on de probabilidad de la variable X. Comprobar que tiene una distribuci on geo-
metrica.
Problema 267 Tenemos una urna con 12 bolas numeradas de 1 a 12. Extraemos dos bolas y
denotamos por X
1
y X
2
los valores que observamos en la primera y en la segunda extracci on.
Sea X la variable denida como el m aximo de las dos extracciones. Se pide:
1. La funci on de distribuci on de la variable X si suponemos que las dos extracciones se
realizan con reemplazamiento.
2. La funci on de distribuci on de la variable X si suponemos que no hay reemplazamiento
entre las extracciones sucesivas.
3. La media de la variable X en las dos situaciones anteriores.
Segundo parcial: problemas 3, 4, 5 y 6.
Examen nal de toda la asignatura: problemas 1, 2, 4 y 5.
5.4.2. Valenciano
Problema 268 La variable aleat` oria X, denida en linterval en [0, 2], te per densitat la recta
que pasa per (2,0) amb pendent negativa. Obtenir f(x) i calcular P(|XE(X)| 1/2). Quina
cota obtindrem per aquesta probabilitat si empr` arem la desigualtat de Chebychev?
Problema 269 A una classe hi ha n estudiants. Un dells coneix un rumor que conta a un
company triat a latzar. Aquest segon estudiant el conta a un tercer, distint del que lil va contar
a ell i triat tambe a latzar. El rumor continua propagant-se daquesta forma: un estudiant el
conta a un altre triat a latzar i diferent del que lil va contar a ell. Quina es la probabilitat de
que en contar-lo k voltes el rumor no haja tornat a cap dels estudiants que ja el coneixien?
Sugger`encia.- Deniu els esdeveniments A
i
={el rumor no es repeteix a cap dels que ja el
coneixen quan es conta per i-`esima volta}, i = 1, . . . , k. Aquests esdeveniments ajuden a denir
lesdeveniment que ens interessa
Problema 270 La variable aleat` oria X te una distribuci o exponencial amb par` ametre Y , que
es al seu torn una variable aleat` oria uniforme sobre [1, 4]. Obtenir la distribuci o conjunta de X
i Y i la esperanca i la varianca de X.
50 CAP

ITULO 5. EX

AMENES PREVIOS
Problema 271 Saproximen 20 n umeros mitjancant lenter mes proper i despres se sumem.
Suposem que els errors comesos s on independents i uniformes en linterval [
1
2
,
1
2
]. Volem
obtenir la probabilitat de que la suma obtinguda sallunye de la suma dels 20 n umeros originals
mes de 3 unitats.
Problema 272 Considerem el seg uent proces:
1. Generem U amb distribuci o uniforme en [0, 1].
2. Denim Y =
1

ln(1 U), on ln es el logaritme neperi` a i una constant positiva.


3. Prenim X = [Y ], la part entera per exces d[Y ].
Es demana:
1. La distribuci o de probabilitat de la variable aleat` oria Y .
2. La funci o de probabilitat de la variable X. Comprovar que te una distribuci o geom`etrica.
Problema 273 En una urna hi han 12 boles numerades de l1 al 12. En tirem fora dues i
designem per X
1
i X
2
els valors observats a la primera i a la segona extracci o, i per X el
m` axim de totes deus extraccions. Es demana:
1. La funci o de distribuci o de la variable X si les dues extraccions s on fetes amb reem-
placament.
2. La funci o de distribuci o de la variable X si les dues extraccions s on fetes sense reem-
placament.
3. La mitjana de la variable X en tots dos casos.
Segon parcial: problemes 3, 4, 5 i 6.
Examen nal de tota la mat`eria: problemes 1, 2, 4 i 5.
5.5. 9 de febrero de 2005
5.5.1. Castellano
Problema 274 Cu al es la probabilidad de que una mano de p oquer contenga s olo una pareja?
Nota: una baraja de p oquer tiene cuatro palos y de cada palo hay 13 cartas. En una mano se
sirven cinco cartas.
Problema 275 Una urna contiene n papeletas numeradas de 1 a n inclusive. Extraemos r al
azar. Sea X el n umero mayor obtenido si las papeletas se reemplazan despues de cada extracci on
y sea Y el n umero mayor si las papeletas no se reemplazan en la urna. Determinar las funciones
de distribuci on, las funciones de cuanta (o probabilidad) y demostrar que
F
Y
(k) < F
X
(k) para 0 < k < n. (5.1)
Problema 276 Calcular la probabilidad de poder formar un tri angulo con dos puntos elegidos
en el intervalo [0, 1] seg un el metodo siguiente: Elegimos un punto al azar y a continuaci on uno
de los trozos, elegido al azar, lo dividimos en dos partes iguales. Calcular, para este metodo, la
probabilidad de que el tri angulo sea obtus angulo.
Nota: dado un tri angulo cuyos lados miden a, b y c y cuyos angulos opuestos son A, B y C,
respectivamente, se verica que c
2
= a
2
+b
2
2ab cos C.
5.6. 21 DE JUNIO DE 2005 51
Problema 277 Probar que para cualquier funci on de densidad de probabilidad se verica
lm
x+
x
_
+
x
1
z
f(z)dz = 0.
5.5.2. Valenciano
Problema 278 Quina es la probabilitat de que una m` a de p` oquer tinga sols una parella?
Nota: una baralla de p` oquer te quatre pals i de cada pal hi han 13 cartes. En una m` a sen
serveixen 5.
Problema 279 Una urna conte n paperetes numerades de 1 a n tots dos inclosos. En traiem r
a latzar. Siga X el major n umero tret si les extraccions han estat fetes amb reempla cament, i
siga Y el major n umero quan les extraccions han estat fetes sense reempla cament. Determinar
les funcions de distribuci o, les funcions de quantia (o probabilitat) i demostrar que
F
Y
(k) < F
X
(k) per a 0 < k < n. (5.2)
Problema 280 Calcular la probabilitat de poder formar un triangle amb els tres segments
obtinguts en triar dos punts a linterval [0, 1] dacord amb el seg uent m`etode: triem un punt a
latzar i tot seguit un dels trossos, tambe triat a latzar, el dividim en dos parts iguals. Calcular
tambe la probabilitat de que el triangle siga obtusangle.
Nota: per a un triangle de costats a, b i c amb angles oposats A, B i C respectivament es
verica que c
2
= a
2
+b
2
2ab cos C.
Problema 281 Probar que per a qualsevol funci o de densitat de probabilitat es cert que
lm
x+
x
_
+
x
1
z
f(z)dz = 0.
5.6. 21 de junio de 2005
5.6.1. Castellano
Problema 282 En una urna hay una bola roja. Extraemos tres cartas de una baraja france-
sa(52 cartas repartidas en 4 palos) y a nadimos a la urna tantas bolas verdes como ases hayamos
extrado. A continuaci on lanzamos 2 veces una moneda cuya probabilidad de cara es p = 1/5
y a nadimos tantas bolas rojas como cruces hayamos obtenido. Finalmente llevamos a cabo 2
extracciones con reemplazamiento de la urna. Si X es el n umero de bolas verdes a nadidas a la
urna e Y el n umero de bolas rojas a nadidas a la urna,
1. Obtener la funci on de probabilidad de X.
2. Obtener la funci on de probabilidad de Y .
3. Si las dos bolas extradas con reemplazamiento son rojas, cu al es la probabilidad de no
haber obtenido ning un as al extraer las 3 cartas de la baraja francesa?
Problema 283 Sea T U(1/c, 1/c), c > 0, y denamos Y = cT. Para k N, las variables
aleatorias X
k
se denen mediante,
X
k
=
_
_
_
1, si 1 < Y < 1/k;
0, si 1/k Y < 1/k;
1, si 1/k Y < 1.
52 CAP

ITULO 5. EX

AMENES PREVIOS
Demostrar que X
k
converge en ley a la variable aleatoria X con funci on de probabilidad
f
X
(1) = f
X
(1) = 1/2 y f
X
(x) = 0, x / {1, 1}.
Problema 284 Denamos la funci on
f(x) =
_
ax
(s+1)
, si x > r;
0, si x r,
con r, s > 0.
1. Determinar a para que f(x) sea una funci on de densidad de probabilidad.
2. Si la variable aleatoria X tiene por densidad f(x), para qu`e valores de s existir a su
esperanza?
Problema 285 Probar utilizando el Teorema Central del Lmite que
e
n
n

k=0
n
k
k!
n

1
2
.
Problema 286 Sobre un crculo cuyo radio R es aleatorio con funci on de densidad
f
R
(r) =
_

_
r
2
9
, r [0, 3];
0 en el resto,
elegimos un punto al azar. Si X designa la distancia del punto al origen, obtener
1. La funci on de distribuci on y la funci on de densidad de X|R = r.
2. La media y la varianza de X.
Problema 287 Un autob us tiene en su recorrido 15 paradas. Supongamos que en la primera
parada suben 20 personas. Cada una de ellas elige al azar e independientemente de las otras en
cu al de las 14 paradas restantes quiere bajar.
1. Si X
i
es una variable aleatoria que vale 1 si alguna de las personas baja en la parada i y
0 en caso contrario, calcular su distribuci on de probabilidad.
2. Calcular el n umero medio de paradas que debe realizar el autob us para que bajen todos los
pasajeros.
Toda la materia: P1, P2, P3 y P6
Segundo parcial: P2, P4, P5 y P6
5.6.2. Valenciano
Problema 288 Tenim una urna amb una bola roja. Triem a latzar 3 cartes duna baralla fran-
cesa (52 cartes repartides en 4 pals) i afegim a la urna tantes boles verdes com asos hem tret.
Despres llancem 2 vegades una moneda que te una probabilitat p = 1/5 de traure cara i afegim
a la urna tantes boles roges com creus hem obtingut. Finalment fem 2 extraccions amb reem-
placament de la urna. Si X i Y representen el n umeros de boles verdes i roges, respectivamente,
que hem afegit a la urna,
5.6. 21 DE JUNIO DE 2005 53
1. Obtenir la funci o de probabilitat dX.
2. Obtenir la funci o de probabilitat dY .
3. Si las dos boles extretes amb reempla cament son roges, quina es la probabilitat de no haber
obtingut cap as al triar les 3 cartes de la baralla francesa?
Problema 289 Siga T U(1/c, 1/c), c > 0. Denim Y = cT i per a k N denim la
variable X
k
mitjancant,
X
k
=
_
_
_
1, si 1 < Y < 1/k;
0, si 1/k Y < 1/k;
1, si 1/k Y < 1.
Demostrar que X
k
convergeix en llei a la variable aleat` oria X amb funci o de probabilitat
f
X
(1) = f
X
(1) = 1/2 i f
X
(x) = 0, x / {1, 1}.
Problema 290 Denim la funci o
f(x) =
_
ax
(s+1)
, si x > r;
0, si x r,
amb r, s > 0.
1. Determineu a per a que f(x) siga una funci o de densitat de probabilitat.
2. Si la variable aleat` oria X te per densitat f(x), per a quins valors d s existir` a la seua
esperanca?
Problema 291 Emprant el Teorema Central del Lmit proveu que
e
n
n

k=0
n
k
k!
n

1
2
.
Problema 292 Sobre un cercle de radi R, aleatori amb funci o de densitat
f
R
(r) =
_

_
r
2
9
, r [0, 3];
0 fora,
triem un punt a latzar. Si per X designem la dist` ancia del punt a lorigen, obtenir
1. La funci o de distribuci o i la funci o de densitat de X|R = r.
2. La mitjana i la vari` ancia dX.
Problema 293 Un autob us te 15 parades al llarg del seu recorregut. Suposem que en la primera
parada pugen a lautob us 20 persones. Cadascuna delles tria a latzar i independentment de les
altres en quina de les altres 14 parades vol baixar.
1. Si X
i
es una variable aleat` oria que val 1 si alguna de las persons baixa en la parada i i 0
en cas contrari, calcular la seua distribuci o de probabilitat.
2. Calcular la mitjana del n umero de parades que deu fer lautob us per a que baixen tots els
passatgers.
Tota la mat`eria: P1, P2, P3 i P6
Segon parcial: P2, P4, P5 i P6
54 CAP

ITULO 5. EX

AMENES PREVIOS
5.7. 6 de junio de 2006
5.7.1. Castellano
Problema 294 Cuando cierta m aquina est a bien ajustada el 60 % de las piezas que produce
son de calidad alta, el 30 % son de calidad media y el resto de calidad baja. Cuando se desajusta,
en cambio, el 40 % de las piezas que produce son de calidad alta, el 40 % son de calidad media
y el resto de calidad baja.
1. Supongamos que en un momento dado la m aquina est a bien ajustada, cu al es la proba-
bilidad de que si se eligen 6 piezas al azar se obtengan 3 de buena calidad y 2 de calidad
media?
2. Supongamos ahora que la m aquina est a desajustada, cu al es la probabilidad de que si se
eligen 6 piezas al azar se obtengan 2 piezas de cada tipo?
3. Se sabe que esta m aquina est a bien ajustada el 80 % del tiempo pero en este momento los
responsables no saben si lo est a o no. Un estudiante de c alculo de probabilidades propone
que se elijan al azar 5 piezas y que se cuenten cuantas hay de cada tipo. En funci on del
resultado se puede saber en que estado se encuentra la m aquina. Los responsables de la
m aquina aceptan su sugerencia y observan 2 piezas de calidad alta, 2 de calidad media y
una de calidad baja. En que estado es m as verosmil que se encuentre la m aquina?
Problema 295
1. Estudia la convergencia en ley de {X
n
}
n1
siendo X
n
una variable aleatoria uniforme-
mente distribuida en {
1
n
,
2
n
, . . . ,
n1
n
, 1}, para n 1
2. Comprueba que {Y
n
}
n2
converge en probabilidad a una constante, siendo Y
n
=
Xn
log n
,
X
n
U(0, 1) para n 2.
Problema 296 Un jugador juega una cantidad inicial de dinero 1e. En cada partida, con igual
probabilidad, duplica su dinero o lo reduce a la mitad. Despues de jugar n partidas cu al es la
ganancia esperada del jugador?
Problema 297 Consideramos X una variable aleatoria con distribuci on uniforme en el inter-
valo [0, ]. Determinar la funci on de densidad de la variable Y = sen(X).
Problema 298 Sean X, U y W variables aleatorias independientes con distribuci on uniforme
en [0, 1]. Denimos Y = XU+(1X)W. Determinar EY y var(Y ). Tiene Y una distribuci on
uniforme en [0, 1]?
Problema 299 Se ofrece un premio a cualquier persona que cuando lance 600 veces un dado
obtenga un mnimo de 125 veces el n umero 6. Lo intentan 300 individuos. Cu al es la probabi-
lidad de que al menos dos de ellos ganen el premio?
Segundo parcial: problemas 2, 3 5 y 6.
Examen nal de toda la materia: problemas 1, 2, 4 y 5.
5.7. 6 DE JUNIO DE 2006 55
5.7.2. Valenciano
Problema 300 Una m aquina ben ajustada produeix un 60 % de peces de qualitat alta, un 30 %
de qualitat mitjana i un 10 % de qualitat baixa. Si es desajusta, les proporcions s on 40, 40 i 20,
respectivament.
1. Si la m` aquina est` a ben ajustada, quina es la probabilitat de que al triar a latzar 6 peces
3 siguen dalta qualitat i 2 de mitjana?
2. Si la m` aquina est` a desajustada, quina es la probabilitat de que al triar a latzar 6 peces
nhi hagen 2 de cada tipus?
3. En un moment donat no sabem si la m` aquina est` a o no ajustada, tot i que el 80 % del
temps ho est` a. Per a saber-ho alg u ens proposa triar 5 peces a latzar i veure quantes
nhi han de cada tipus i decidir en funci o del resultat. Feta la prova trobem 2 peces dalta
qualitat i 2 de mitjana. En quin es mes verosmil que es trobe la m` aquina?
Problema 301 1. Estudia la converg`encia en llei de {X
n
}
n1
on X
n
una variable aleat` oria
uniformemente distribuda en {
1
n
,
2
n
, . . . ,
n1
n
, 1}, n 1
2. Comprova que {Y
n
}
n2
convergeix en probabilitat a una constant, essent Y
n
=
Xn
log n
,
X
n
U(0, 1) n 2.
Problema 302 Un jugador juga una quantitat inicial c. En cada partida, amb igual probabi-
litat, la quantitat jugada es duplica o es redueix a la meitat. Despres de jugar n partides, quin
es el guany esperat del jugador?
Problema 303 Considerem X una variable aleat` oria amb distribuci o uniforme en linterval
[0, ]. Determinar la funci o de densitat de la variable Y = sin(X).
Problema 304 Les variables aleat` ories X, U i W s on independents i totes tres U(0, 1). Denim
Y = XU + (1 X)W. Determinar E(Y ) i var(Y ). Te Y una distribuci o U(0, 1)?
Problema 305 Sofereix un premi a qualsevol que en 600 llancaments dun dau obtinga un
mnim de 125 sisos. Ho intentan 300 individus. Quina es la probabilitat de que al menys dos
deells guanyen el premi?
Segon parcial: problemes 2, 3 5 i 6.
Examen nal de tota la mat`eria: problemes 1, 2, 4 i 5.

También podría gustarte